Download as pdf or txt
Download as pdf or txt
You are on page 1of 64

CUPRINS

Algebraic Geometric Equation........................................................................................................................ 2


An Angle Inequality in Simple Polygons.............................................................................................................. 2
An Inequality via Youngs Theorem 3
CUM CONSTRUIM PATRATE MAGICE DE ORICE ORDIN ... 5
PROBLEME IN ENGLEZ ..................................................... 7
THE MATHEMATICAL GRAMMAR SCHOO LCUP - MATHEMATICS - BELGRADE, June23, 2015............. 10
Concursul MATHEMATICAL GRAMMAR SCHOOL CUP - MATEMATIKA - BEOGRAD, 23. JUN 2015. 16
Test tip BAC M2. 19
PROBLEME PROPUSE ...................................................................................................................................... 20
CLASA A V-A ....................................................................................................................................................... 20
Setul I ................................................................................................................................................................... 20
Setul II .................................................................................................................................................................. 20
CLASA A VI-A ...................................................................................................................................................... 21
Setul I ................................................................................................................................................................... 21
Setul II .................................................................................................................................................................. 23
CLASA A VII-A ..................................................................................................................................................... 25
Setul I ................................................................................................................................................................... 25
Setul II .................................................................................................................................................................. 26
CLASA A VIII-A .................................................................................................................................................... 28
Setul I ................................................................................................................................................................... 28
Setul II .................................................................................................................................................................. 29
CLASA A IX-A ...................................................................................................................................................... 30
Setul I ................................................................................................................................................................... 30
Setul II .................................................................................................................................................................. 31
CLASA A X-A ....................................................................................................................................................... 32
Setul I ................................................................................................................................................................... 32
Setul II .................................................................................................................................................................. 33
CLASA A XI-A ...................................................................................................................................................... 34
Setul I ................................................................................................................................................................... 34
Setul II .................................................................................................................................................................. 35
CLASA A XII-A ..................................................................................................................................................... 37
Setul I ................................................................................................................................................................... 37
Setul II .................................................................................................................................................................. 38
INDICAII I RSPUNSURI ............................................................................................................................... 40
PROBLEMS IN ENGLISH................................................................................................................................. 40
CLASA A V-A ....................................................................................................................................................... 40
CLASA A VI-A ...................................................................................................................................................... 42
CLASA A VII-A ..................................................................................................................................................... 44
CLASA A VIII-A .................................................................................................................................................... 47
CLASA A IX-A ...................................................................................................................................................... 49
CLASA A X-A ....................................................................................................................................................... 50
CLASA A XI-A ...................................................................................................................................................... 52
CLASA A XII-A ..................................................................................................................................................... 55
RUBRICA REZOLVITORILOR ............................................................................................................................ 57
ANUN CONCURS................................................................................. 59
ANUN SIMPOZION ............................................................. 61

Articole metodico-tiinifice_______________________________________________________________________________2

Algebraic Geometric Equation


Claudia Nanuti -Colegiul Naional Economic Theodor Costescu
Solve in real numbers:

Solution
The

problem

admits

simple

geometric

interpretation. Let
. Let

in

the

Cartesian coordinates
be an arbitrary point in the

plain. We are requested to solve the equation:


or simpler,

Note

that

by

the

triangle
inequality,
and
which tells us that the required equality may be satisfied only when both ,
and
. The first is satisfied with the points of the segment
, the second with the points of the segment
, implying that the solution to the problem lies at the
intersection of the two segments. Observe that
is a rectangle, with the diagonals
and
that
intersect at the center of the rectangle. It was that

is the only solution to the given equation.

An Angle Inequality in Simple Polygons


Daniel Sitaru,Colegiul Naional Economic Theodor Costescu
Let

, be a simple (without self intersections) polygon, with the inner angles


Then

Examples
In a triangle with angles

In a simple quadrilateral with angles

Nr. 2/2015 ____________________________________________________________________ Revista de matematic Minimath

3_______________________________________________________________________ Articole metodico-tiinifice

Proof:
Define

(which is known to be

). By the Geometric Mean - Arithmetic Mean inequality,

Implying the Binomial expansion,

An Inequality via Youngs Theorem


Daniel Sitaru,Colegiul Naional Economic Theodor Costescu
For

the following inequality holds

The equality is reached for

The above inequality follows from Youngs theorem:


Let be a continuous strictly increasing function on an interval
and
,

with the equality is only achieved if

and

it inverse. For

Revista de matematic Minimath____________________________________________________________________ Nr. 2/2015

Articole metodico-tiinifice_______________________________________________________________________________4

Proof of Sitarus inequality


Choose
Youngs inequality for any

so that
.

. Obviously such

satisfies the conditions of

Now by elementary calculus

and also

By Youngs theorem it follows that

In other words,

By replacing

with

and

with

we obtain

.
Similarly,
and
Adding the three proves the desired inequality.
Clearly, the statement and the proof extend to any number of parameters:
For positive

Nr. 2/2015 ____________________________________________________________________ Revista de matematic Minimath

5_______________________________________________________________________ Articole metodico-tiinifice

CUM CONSTRUIM PATRATE MAGICE DE ORICE ORDIN


Prof.Magdalena Ptuleanu.Dr.Tr.Severin
Cred c ai ntlnit de multe ori n reviste un tablou ptrat de numere, n care suma numerelor de pe
oricare linie s fie egal cu aceea de pe oricare coloan i cu aceea de pe cele
dou diagonale. Acesta este un ptrat aritmetic sau magic! Cel mai simplu exemplu este cel pentru ordinul 3,
cu numere consecutive de la 1 la 32.
67 1 43
Constanta magic e: CM = (1+n2)*n2/2 . n cazul n=3,CM = 15
2 7 6
13 37 61
Dac am alege numere prime, CM = 67+1+43=111 .
9 5 1
31 73 7
Ptratele magice sunt cunoscute din Antichitate .
4 3 8
In Orientul ndepartat erau considerate de ctre astrologi cu influen binefctoare, fiind
imprimate pe tbli de metal i purtate ca amulete . Din secolul VII au nceput s fie studiate de matematicieni
, pn spre Renatere i n Europa Occidental . Matematicianul Philippe de la Hire (1640-1718) a gsit la
Paris un manuscris grecesc din secolul XIV, aparinnd lui Manuel Moscopoulos, care a scris o metoda de
generare a ptratelor magice pentru n impar i par .
Prima metod pentru n impar a fost dat n 1612 de matematicianul Bachet de Meziriac (1581-1638),
mbuntit de ctre Ph.de la Hire, astfel nct s genereze toate ptratele magice de acel ordin . Se
deseneaz ptratele cu 9 sau 25 csue (pentru n=3 sau 5), apoi se adaug pe fiecare latur exterioar a lor
n form de scar, alte ptrate de aceeai mrime, rezultnd trei oblice paralele cu diagonalele principal i
secundar din matricea creat prin aceast bordare.Toate elementele adugate, mai puin cele existente n
coluri se trec peste cel central n locurile libere rmase pe orizontal i vertical, dup cum se vede i din cele
dou figuri alturate .
Analog pentru ptratele magice de ordin par, Philippe de la
3
3
Hire
a gsit o regul general pentru ordinul dublu par pstrand
2
6
2
7
6
ca
punct de plecare dou iruri : 1...4n respectiv 0...(n2 n4 ) .
1
5
9
1 9 5 1 9 Pentru n=4 incepem cu irurile 1 2 3 4
4
8
4 3 8
0 4 8 12
formm cu ele dou ptrate aritmetice cu
7
7
constantele magice CM1 = 4*(1+4)/2 i CM2 = 4*(0+12)/2 .
Se face ptratul magic respectnd proprietatea de adunare a dou ptrate magice ajuttoare .
n prima linie alegem numerele ntmpltor astfel nct s fie echidistante n progresia aritmetic cu raia
1 i s se aeze simetric fa de verticala median . Regula se repet i la al doilea ptrat aplicnd regula la
numerele din coloana nti .Apoi pe linia a doua a primului ptrat punem numerele primei linii aezate invers .
Transcriem ultimele dou linii simetrice cu primele dou fa de orizontala median. Aceeai regul se aplic
pentru coloane n ptratul al doilea, obinndu-se ptratul magic dup cum se vede n figurile urmtoare :
4 3 2 1
12 0
0 12
16 3
2
13
1 2 3 4
4 8
8 4
5
10
11 8
1

3 4

12

2 1

12

15

14

Construim un ptrat magic de ordin impar, cu elemente consecutive 1..n*n. Toate se pot obine permutnd
liniile, coloanele sau diagonalele sau cu procedeul de mai jos.
program un_ptrat_magic; uses crt ; label 1;
var a :array[1..20,1..20] of integer ; var I,j,k,n:integer;
Begin
1:clrscr; Write(dati dimensiunea ptratului); read(n);
if (n>19) or (n/2=n div 2 ) then go to 1 ;
i:=(n+1) div 2 ; j:=n+1;
for k:=1 to n*n do
begin if k-(k div n )*n=1 then j:=j-1
else begin i:=i+1;if i>n then i:=1;
Revista de matematic Minimath____________________________________________________________________ Nr. 2/2015

Articole metodico-tiinifice_______________________________________________________________________________6

j:=j+1; if j>n then j:=1;


end;
a[i,j]:=k;
write(i=,i, j=,j); writeln( a[i,j]=,a[i,j]);
end; writeln; writeln(PATRATUL MAGIC ESTE : ); writeln;
for i:=1 to n do
begin for j:=1 to n do write(a[I,j], ); writeln;
end; repeat until keypressed;
END.
n continuare, programul pentru construirea tuturor ptratelor magice n impar.
Program toate_ptratele_magice; uses crt;
Var i,j,x,y,m,a1,a2,b1,b2,n,nrpatrate:integer ;
a,b,c:array[1..20,1..20] of integer;
BEGIN clrscr;Writeln(dati ordinul patratului magic(nr. Impar si n>=5) );read(n);
if (n<5) of not(odd(n)) then writeln(nu genereaza patrate magice ! );
else writeln(cm=n*(n*n+1)/2=,n*(n*n+1)/2:8:2);
delay(3000); nrpatrate:=0;writeln(PATRATELE MAGICE SUNT:);
for x:=1 to n-1 do begin for y:=1 to n-1 do
begin Ifx<>y then begin a1:=0;a2:=0;b1:=0;b2:=0;
for i:=1 to n do
for j:=1 to n do
begin
a[I,j]:=(x*(i-1)+j-1) mod n ;
b[I,j]:=(y*(i-1)+j-1) mod n;
if i=j then begin a1;=a1+a[I,j]; b1:=b1+b[I,j]; end;
if I=n-j+1 then begin a2:=a2+a[I,j];b2:=b2+b[I,j]; end;
end;
m:=n*(n-1) div 2 ;
if (a1=m) and (a2=m) and (b1=m) and (b2=m) then
begin
for i:=1 to n do
begin
for j:=1 to n do
begin c[I,j]:=a[I,j]*n+b[I,j]+1; write(c[i,j], );end;
writeln; nrpatrate:=nrpatrate+1;
end;
write(nrpatrate=,nrpatrate/n:8:2);
end; end; end;
end; repeat until keypressed;END.
* Programele sunt construite original dup procedeul matematic descris de De La Hire.
Bibliografie(teoretica) Aplicaii ale matematicii Fl. T. Campan, Colectia Alfa, Buc. 1984

Nr. 2/2015 ____________________________________________________________________ Revista de matematic Minimath

7_______________________________________________________________________ Articole metodico-tiinifice

PROBLEME IN ENGLEZ
1. S se demonstreze c:
Daniel Sitaru, Drobeta Turnu Severin
Soluie:
Inegalitatea se scrie succesiv:

Demonstrm prin inducie dup

c:

Verificare:

sau
se presupune adevrat
de demonstrat

Artm c:

Din
deoarece
n inegalitatea:

lum

i obinem:

Prove that:

By maths induction:
We take

and obtain:

2. S se calculeze:

Daniel Sitaru, Drobeta Turnu Severin


Soluie:
Demonstrm prin inducie dup

c:

Revista de matematic Minimath____________________________________________________________________ Nr. 2/2015

Articole metodico-tiinifice_______________________________________________________________________________8

Verificare:
se presupune adevrat

de demonstrat

deoarece

By math induction:

On big screens: Rich mouse,Poor mouse!


with Nick Nolte and Peter Strauss

Nr. 2/2015 ____________________________________________________________________ Revista de matematic Minimath

9_______________________________________________________________________ Articole metodico-tiinifice

3. Fie

. S se arate c:
Daniel Sitaru, Drobeta Turnu Severin

Soluie:

Analog:

If
then
Clasa a X-a
1. Prove that if

, and

, then we have that

.
Mihai Babuia, Ph.D

CLASA A XI-a
2. Consider

a sequence of real numbers which is nondecreasing and divergent and consider


to be a solution of the equation
. Prove that

the sequence

is nonincreasing and bounded and compute

3. Find the limit of the sequence


4. Consider

the

sequence
. Compute

.
Mihai Babuia, Ph.D
.

, defined by
defined
.

by

Mihai Babuia, Ph.D


,
Mihai Babuia, Ph.Dr

Rubrica realizat de prof. Dorin Marghidanu, Corabia, Olt

Revista de matematic Minimath____________________________________________________________________ Nr. 2/2015

Articole metodico-tiinifice_______________________________________________________________________________10

THE MATHEMATICAL GRAMMAR SCHOO LCUP


- MATHEMATICS BELGRADE, June23, 2015

PART ONE
Problems 1 to 8 are multiple choice problems. Out of five oered choices for a problem, exactly one is the
correct answer. On the answer sheet you should circle only the letter that corresponds to the answer you
have chosen.
33

1. What is the last digit of the number 333 ?


(A) 1;

(B) 3;

(C) 5;

(D) 7;

(E) 9.
2

2. If x and y are positive real numbers such that x + y = 87 and xy=30, then their sum x+y is equal to:
(A) 37;

(B) 87; (C) 73;

3. If numerator a of the quotient

(D) 85;

(E) 102.

increases 20%, and the denominator b decreases

20%, by how many percent will the quotient increase?


(A) 40%;
(B) 20%; (C) 25%;
(D) 36%;

(E) 50%.

4. The equation |2x+5| = x+5 has:


(A) exactly one solution and it is negative;
(B) exactly one solution and it is positive;
(C) exactly one solution and it is equal to zero; (D) two solutions; (E) no solutions.
5. The areas of three faces of a cuboid (rectangular parallelepiped) are equal to 12, 8, and 6. The volume of
that cuboid is:
(A) 24;

(B) 26;

(C) 42;

(D) 44;

(E) 48.

6. The height lengths of a triangle are equal to 12, 15, and 20.The area of that triangle is equal to:
(A) 84;

(B) 90;

(C) 120;

(D) 150;

(E) 180.
n

7. The number of all pairs (m,n) of positive integers m and n such that(m - 8)(m - 10) = 2 is equal to:
(A) zero;

(B) one;

(C) two;

(D) three;

(E) four or more.

8. In a right-angled trapezoid ABCD, with right angle at the vertex A, the diagonals AC and BD are
perpendicular to each other. If the bases of the trapezoid are AB=9 and CD=4, the length of the side AD is
equal to:
(A) 5;

(B) 52;

(C) 6;

(D) 53;

(E) 8.

PART TWO
9. Find all positive integers n such that the last digit (in the decimal representation) of the
sum 1 + 2 + 3 + + n is equal to7.
10.
Let A1, B1 and C1 be the touching points of the inscribed circle of a triangle ABC with the sides BC=a,
AC=b and AB=c, respectively, and let AC1=p, BA1=q and CB1=r. Prove that the following in equality holds:

Nr. 2/2015 ____________________________________________________________________ Revista de matematic Minimath

11_______________________________________________________________________ Articole metodico-tiinifice

When does the equality hold?


11. Show that there is a number of the form 77...77 which is divisible by 2017.
nsevens
12. Mariana decided to study for 23 days in June and to relax during the rest of the days. In how many way
scan Mariana choose the seven days to relax if she decided never to relax for two days in a row?

PART ONE

The correct answers are: 1. (B) 2. (C) 3. (E) 4. (D) 5. (A) 6. (D) 7. (C) 8. (C)
PART TWO
1. Find all positive integers n such that the last digit (in the decimal representation) of the
sum 1 + 2 + 3 + + n is equal to 7.
Solution. Recall that 1 + 2 + + n =

. If the last digit of

is 7, then the last digit of n(n+1) has to

be equal to 4. Now we have the following possibilities:


the last digit of n
the last digit of n+1
the last digit of n(n+1)

0
1
0

1
2
2

2
3
6

3
4
2

4
5
0

5
6
0

6
7
2

7
8
6

8
9
2

9
0
0

We see from the table that the last digit of the product of two consecutive positive integers cannot be equal to
4, which means that there is no solution to this problem.
2. Let
and be the touching points of the inscribed circle of a triangle ABC with its sides BC= a, AC=
b, and AB= c, respectively, and let AC1= p, BA1= q, and CB1= r. Prove that the following inequality holds:

When does the equality hold?


C

p C1

Solution. It is well known that

If we rewrite our inequality, we see that we ought to prove that

Revista de matematic Minimath____________________________________________________________________ Nr. 2/2015

Articole metodico-tiinifice_______________________________________________________________________________12

The left hand side of the inequality is equal to

Using the fact that, for

(here the equality holds if and only if t = 1) we get

It is now easy to see that

if and only if

,
i.e., if and only if ABC is an equilateral triangle.
3. Show that there is a number of the form 77...77 which is divisible by 2017.
nsevens
Solution1. We will prove that an even stronger statement is true: one of the numbers that can be written using
at most 2017 sevens is divisible by 2017. Assume the contrary is true, i.e., that none of the numbers from
the set
{7,77,777,...,77...77 }
2017
is divisible by 2017. Since we have 2016 possible non-zero remainders when a number is divided by 2017,
by using the Pigeon-Hole Principle, we conclude that at least two of these numbers have the same
remainder. Let those be
ni=77...77 and nj=77...77 ,where i<j.
I sevens j-sevens
We now have that 2017 | njni. In other words, for some integer k,
i

2017k= njni=77...70...00 =10 77...7.


jI I

ji

Since gcd(2017,10) =1,we conclude that 2017 | 77...7.


j i
Solution2.Since 2017 is a prime number, by the Little Fermats Theorem, 10

2016

20171.

Therefore,
9 77...7 =9 7 (10

2015

2014

+10

2016

+ +10+1) =7(10

1) =2017k, k N.

2016
Since gcd(2017,9) = 1, we get that 2017 | 77...7.
2016
4. Mariana decided to study for 23 days in June and to relax during the rest of the days. In how many ways
can Mariana choose the seven days to relax if she decided never to relax for two days in a row?
Solution.Let1 d1<d2< <d7 30 be one choice of days satisfying the conditions.
Nr. 2/2015 ____________________________________________________________________ Revista de matematic Minimath

13_______________________________________________________________________ Articole metodico-tiinifice

Note that the requirement that there are no two consecutive numbers among the djsand 1 dj 30 for all j is
equivalent to the requirement that
1 d1 < d21 < d32 < d43 < d54 < d65 < d76 306 = 24.
In other words, choosing days 1 d1<d2< <d7 30 satisfying the conditions of our problem is the same as

choosing any seven numbers d 1 = d1,d 2 = d21,...,d k = dkk+1,...,d 7 = d76 from the set {1,2,3,...,23,24}.
This can be done in
= dierent ways.
Note. We divide by 7! because there is a unique way to arrange seven dierent chosen numbers in an

increasing sequence d 1,d 2,...,d 7.

Concursul MATHEMATICAL GRAMMAR SCHOOL CUP


- MATEMATICA BELGRAD, 23 IUNIE , 2015

PARTEA INTAI
Problemele de la 1 la 8 sunt in sistem grila. Din cele 5 raspunsuri ale unei probleme exact unul este corect.
Pe randul cu raspunsuri incercuiti litera care corespunde raspunsului ales.
33

1. Care este ultima cifra a numarului 333 ?


(A) 1;

(B) 3;

(C) 5;

(D) 7;

(E) 9.
2

2. Daca x si y sunt numere reale pozitive astfel incat x + y = 87 si xy=30 atunci suma lor x+y este egala cu :
(A) 37;

(B) 87; (C) 73;

3.Daca numaratorul a al fractiei


20%,cu ce procent va creste fractia ?
(A) 40%;
(B) 20%; (C) 25%;

(D) 85;

(E) 102.

creste cu 20%, si numitorul b descreste cu 20%


(D) 36%;

(E) 50%.

4. Ecuatia |2x+5| = x+5 are :


(A) exact o solutie si ea e negativa ;
(B)exact o solutie si ea e pozitiva ;
(C)exact o solutie si ea este egala cu 0 ; (D)doua solutii ; (E) nicio solutie .
5. Ariile a trei fete ale unui cuboid (paralelipiped dreptunghic) sunt egale cu 12, 8, si 6. Volumul cuboidului
este:
(A) 24;

(B) 26;

(C) 42;

(D) 44;

(E) 48.

6. Lungimile inaltimilor unui triunghi sunt egale cu 12, 15, si 20. Aria triunghiului este egala cu:
(A) 84;

(B) 90;

(C) 120;

(D) 150;

(E) 180.
n

7. Numarul tuturor perechilor de numere naturale (m,n) astfel incat (m - 8)(m - 10) = 2 este egal cu :
(A) zero;

(B) unu;

(C) doi;

(D) trei; (E)patru sau mai multe .

8. In trapezul dreptunghic ABCD, cu un unghi drept in A, diagonalele AC si BD sunt perpendiculare .Daca


bazele trapezului sunt AB=9 si CD=4,lungimea laturii AD este egala cu :
(A) 5;

(B) 52;

(C) 6;

(D) 53;

(E) 8.

Revista de matematic Minimath____________________________________________________________________ Nr. 2/2015

Articole metodico-tiinifice_______________________________________________________________________________14

Raspunsurile corecte sunt : 1. (B) 2. (C) 3. (E) 4. (D) 5. (A) 6. (D) 7. (C) 8. (C)

PARTEA A DOUA
9.Gasiti toate numerele naturale n pentru care ultima cifra( in reprezentarea zecimala ) a sumei 1 + 2 + 3 +
+ n este egala cu 7.
Solutie . Sa ne reamintim ca 1 + 2 + + n =

.Daca ultima cifra a lui

este 7, atunci ultima cifra a

lui n(n+1) trebuie sa fie egala cu 4. Avem urmatoarele posibilitati:


Ultima cifra a lui n
0
Ultima cifra a lui n+1
1
Ultima cifra a lui n(n+1) 0

1
2
2

2
3
6

3
4
2

4
5
0

5
6
0

6
7
2

7
8
6

8
9
2

9
0
0

Observam din tabel ca ultima cifra a produsului a doua numere naturale consecutive nu poate fie egala cu 4,
ceea ce inseamna ca problema nu are solutie .
10.
Fie
si punctele de intersectie ale cercului inscris in triunghiul ABC cu laturile sale BC= a,
AC= b, si AB= c, si sa notam lungimile AC1= p, BA1= q, si CB1= r. Demonstrati urmatoarea inegalitate :

Cand are loc egalitatea?


C

p C1

Solutie. Este stiut ca

Daca rescriem inegalitatea noastra vedem ca avem de demonstrat ca

Membrul stang al inegalitatii este egal cu

Folosind faptul ca , pentru orice

(egalitatea are loc aici daca si numai daca t = 1) obtinem

Acum e usor de vazut ca

Nr. 2/2015 ____________________________________________________________________ Revista de matematic Minimath

15_______________________________________________________________________ Articole metodico-tiinifice

daca si numai daca

adica daca si numai daca ABCeste triunghi echilateral .


11. Aratati ca exista un numar de forma 77...77 care este divizibil prin 2017.
n cifre 7
Solutia 1. Vom demonstra chiar mai mult: unul dintre numerele care pot fi scrise folosind cel mult 2017 cifre 7
este divizibil cu 2017. Sa presupunem contrariul , adica faptul ca niciunul dintre numerele din multimea
{7,77,777,...,77...77 }
2017
nu este divizibil cu 2017. Deoarece exista 2016 resturi posibile nenule la impartirea la 2017, folosind
principiul cutiei deducem ca macar doua din aceste numere dau acelasi rest la impartirea la 2017. Sa le
notam
ni=77...77 si nj=77...77 ,unde i<j.
i cifre 7 j-cifre 7
Avem ca 2017 | njni. Altfel , pentru un anume numar natural k :
i

2017k= njni=77...70...00 =10 77...7.


ji i

ji

Deoarece c.m.m.d.c. (2017,10) =1 deducem ca 2017 | 77...7.


j i
Solutia 2. Deoarece 2017 este numar prim , din Mica teorema a lui Fermat rezulta ca 10

2016

1(mod 2017) .

Apoi
9 77...7 =9 7 (10

2015

2014

+10

2016

+ +10+1) =7(10

1) =2017k, k N.

2016
Deoarece c.m.m.d.c. (2017,9) = 1 obtinem ca 2017 | 77...7.
2016
12. Mariana decide sa studieze 23 zile in iunie si sa se relaxeze in cele ramase.In cate feluri poate alege
Mariana cele 7 zile de relaxare daca ea hotaraste sa nu se relaxeze doua zile la rand ?
Solutia.Fie 1 d1<d2< <d7 30 o alegere a zilelor ca in ipoteza.
Observam ca ipoteza de a nu exista doua numere consecutive printre cele notate dj unde 1 dj 30 pentru
toti j este echivalenta cu cerinta ca
1 d1 < d21 < d32 < d43 < d54 < d65 < d76 306 = 24.
Altfel alegerea zilelor 1 d1<d2< <d7 30 satisfacand conditiile ipotezei este echivalenta cu alegerea

arbitrara a 7 numere d 1 = d1,d 2 = d21,...,d k = dkk+1,...,d 7 = d76 din multimea {1,2,3,...,23,24}. Acest lucru
poate fi facut in
modalitati diferite.
Observatie . Impartim prin 7! deoarece exista un singur mod de a aranja 7 numere diferite intr-un sir

crescator d 1,d 2,...,d 7.

Revista de matematic Minimath____________________________________________________________________ Nr. 2/2015

Articole metodico-tiinifice_______________________________________________________________________________16

Concursul MATHEMATICAL GRAMMAR SCHOOL CUP


- MATEMATIKA BEOGRAD, 23. JUN 2015.

PRVI DEO
U zadacima od 1 do 8 treba odrediti taan odgovor. Od 5 datih odgovora samo je jedan taan. Zaokruiti
slovo koje odgovara tanom odgovoru.
33

1. Koja je poslednja cifra broja 333 ?


(A) 1;

(B) 3;

(C) 5;

(D) 7;

(E) 9.
2

2. Ako su x i y realni pozitivni brojevi tako da je x + y = 87 i xy=30 je njihov zbir x+y jednak :
(A) 37;

(B) 87; (C) 73;

(D) 85;

(E) 102.

3.Ako se brojilac a, razlomka

uveava za 20%, a imenilac b se smanjuje za 20% ,


za koliko procenta e se uveati razlomak ?
(A) 40%;
(B) 20%; (C) 25%;
(D) 36%;
(E) 50%.

4. Jednaina |2x+5| = x+5 ima :


(A) tano jedno negativno reenje ;(B) tano jedno pozitivno reenje;
(C) tano jedno reenje jednako 0 ;
(D) dva reenja ; (E) nema reenje .
5. Povrine triju strana kvadra (pravouglog paralelopipeda ) su 12, 8, i 6. Zapremina kvadra je:
(A) 24;

(B) 26;

(C) 42;

(D) 44;

(E) 48.

6. Duine visina trougla su 12, 15, i 20. Povrina trogla je :


(A) 84;

(B) 90;

(C) 120;

(D) 150;

(E) 180.
n

7. Broj svih ureenih parova prirodnih brojeva (m,n), takvih da je (m - 8)(m - 10) = 2 je:
(A) nula;

(B) jedan;

(C) dva;

(D) tri; (E) etiri ili vie .

8. Dat je pravougli trapez ABCD, sa pravim uglom kod temena A i normalnim diagonalama AC i BD . Ako su
osnove trapeze AB=9 i CD=4, duina stranice AD je :
(A) 5;

(B) 52;

(C) 6;

(D) 53;

(E) 8.

Tani odgovori su : 1. (B) 2. (C) 3. (E) 4. (D) 5. (A) 6. (D) 7. (C) 8. (C)

Nr. 2/2015 ____________________________________________________________________ Revista de matematic Minimath

17_______________________________________________________________________ Articole metodico-tiinifice

DRUGI DEO
9.Nai sve prirodne brojeve n za koje je poslednja cifra (u decimalnom zapisu) zbira 1 + 2 + 3 + + n jednaka
7.
Reenje . Da se podsetimo da je 1 + 2 + + n =

.Ako je poslednja cifra od

7, onda poslednja

cifra od n(n+1) treba da bude 4. Imamo sledee mogunosti:


Poslednja cifra za n
0
Poslednja cifra za n+1 1
Poslednja cifra za n(n+1) 0

1
2
2

2
3
6

3
4
2

4
5
0

5
6
0

6
7
2

7
8
6

8
9
2

9
0
0

Vidi se iz tablice da poslednja cifra proizvoda dva uzastopna prirodna broja ne moe biti 4, prema tome
zadatak nema reenje.
10.
Neka su
i presene take upisanog kruga u trouglul ABC ije su stranice BC= a, AC= b, i
AB= c, obeleiemo duine AC1= p, BA1= q, i CB1= r. Dokazati nejednakost :

Kada vai jednakost?


C

p C1

Reenje .Zna se da je

Ako prepiemo nau nejednakost, vidimo da treba dokazati da je

Leva strana nejednakosti je

Koristei injenicu da za svako

(jednakost vai ako I samo ako je t = 1) dobiemo

Sada je lako videti da je

ako i samo ako je

Revista de matematic Minimath____________________________________________________________________ Nr. 2/2015

Articole metodico-tiinifice_______________________________________________________________________________18

odnosno, ako I samo ako je trougao ABC jednakostranian trougaol .


11. Dokazati da postoji broj oblika 77...77 od n cifara koji je deljiv sa 2017.
Reenje br.1 Dokazaemo vie :jedan od brojeva koji se mogu napisati pomou najvie 2017 cifara 7 je deljiv
sa 2017.Pretpostavimo suprotno, odnosno da nijedan od brojeva iz skupa
{7,77,777,...,77...77 }
2017
nije deljiv sa 2017. Poto postoje 2016 moguih ostataka razliitih od nule pri delenju sa 2017, koristei
princip kutije yaljuujemo da bar dva broja imaju isti ostatak pri delenju sa 2017. Obeleiemo ih sa
ni=77...77 i nj=77...77 ,gde je i<j.
i cifara 7 j- cifara 7
Imamo da 2017 | njni. Tako da za neki prirodan broj k vai :
i

2017k= njni=77...70...00 =10 77...7.


ji i

ji

Poto je NZD (2017,10) =1 zakljuujemo da 2017 | 77...7.


j i
Reenje br.2 Poto je 2017 prost broj ,iz Fermatove male teoreme proizilazi da je 10

2016

1(mod 2017) .

Onda
9 77...7 =9 7 (10

2015

2014

+10

2016

+ +10+1) =7(10

1) =2017k, k N.

2016
Poto je NZD (2017,9) = 1 sledi da 2017 | 77...7.
2016
12. Mariana je odluila da ui 23 dana u junu a ostale dane da odmara.Na koliko naina moe Mariana da
izabere onih 7 dana odmora ako je odluila da se ne odmara u dva uzastopna dana ?
Reenje.Neka je 1 d1<d2< <d7 30 jedan nain izbora kao u pretpostavci zadatka.
Konstatujemo da pretpostavka da ne postoje dva uzastopna broja meu datim brojevima obeleenih sa dj
gde je 1 dj 30 za svako j je ekviivalentna uslovu da
1 d1 < d21 < d32 < d43 < d54 < d65 < d76 306 = 24.
Tako, odabir dana 1 d1<d2< <d7 30 koji zadovoljava uslove pretpostavke je ekvivalentan sluajnom

odabiru 7 brojeva d 1 = d1,d 2 = d21,...,d k = dkk+1,...,d 7 = d76 iz skupa {1,2,3,...,23,24}. Ovo se moe
uraditi na
razliitih naina.
Napomena .Podelimo sa 7! zato to postoji samo jedan nain raspodele 7 razliitih brojeva u rastui

niz d 1,d 2,...,d 7.


Rubric realizat de: Prof. Srdjan Ognjianovic, Belgrad, Serbia ,
Prof.dr.Adrian Lupu, Dr. Tr. Severin, Prof. Dojna Ardeljan, Vrsac, Serbia

Nr. 2/2015 ____________________________________________________________________ Revista de matematic Minimath

19_______________________________________________________________________ Articole metodico-tiinifice

Test tip BAC M2


Prof. Rdulescu Valentin
L.T. Constantin Brncoveanu - Scorniceti
Subiectul I 30p
1. Sa se rezolve inecuatia 3
2. Sa se calculeze probabilitatea ca aruncand un zar sa apara fata ce reprezinta un numar prim
3. Sa se rezolve in R ecuatia logaritmica
4. Sa se verifice daca numerele
,
sunt in progresie geometrica.
5. Fie vectorii
.Sa se calculeze lungimea vectorului
6. Daca tgx=2 sa se calculeze
SubiectulII30p
1. Fie matricile
a)Sa se verifice ca AB=
b)Sa se calculeze det(A+B)
c) Sa se resolve ecuatia matriciala AX=B,
2. Se considera polinomul
-2
,cu radacinile
a)Sa se determine catul si restul impartirii lui f la
b)Sa se calculeze + +
c) Sa se gaseasca radacinile reale ale lui f
Subiectul III.30p
1. Se considera functia
a) Sa se scrie ecuatia asimptotei oblice la
b) Sa se calculeze
c) Sa se scrie ecuatia tangentei la graficul lui f in punctul de abcisa x=1
2. Se considera functiile
,F(x)=
a) Sa se determine functia f(x) stiind ca functia F(x) este o primitiva a sa.
b) Sa se demonstreze ca
c) Sa se calculeze

NOTA:Se acorda 10p din oficiu.Nota finala se obtine impartind suma punctelor la 10.

Revista de matematic Minimath____________________________________________________________________ Nr. 2/2015

Probleme propuse_______________________________________________________________________________________ 20

PROBLEME PROPUSE
CLASA A V-A

Setul I
1. Aflai cifrele a, b i c astfel nct:
abc 21 + 1abc = 15321 + abc
Prof. Drghici Ana Maria, Liceul "Constantin Brncoveanu" Horezu
2. a) Cte cifre are numrul 12345678910111213..20152016?
b) Care este cifra de pe locul 2016?
Prof. Ionic Constantin ,Dr.Tr.Severin
3. La o mprire de numere naturale, dempritul este de 55 ori mai mare dect restul, mpritorul este
triplul ctului,iar restul este o treime din ct. Aflai dempritul, mpritorul, ctul i restul.
Prof. Ionic Constantin, Dr.Tr.Severin
4. Se consider mulimile: A={4x+1,4x+3,4x+5,5x-3}i B={4x+2,6x,2016x-14080}. S se determine
valorile lui x N pentru care A B
Prof. Ionic Constantin, Dr.Tr.Severin
5. Adugai la dreapta numrului 3412 dou cifre astfel nct numrul obinut s fie divizibil cu
numerele 4 , 5 i 11.
Prof. Ionic Constantin
6. Ce numr cuprins ntre 10000 i 11000 se mparte exact cu oricare din numerele 10,12,14,16,18,20?
Prof. Ionic Constantin
7. Determinai numrul natural x, tiind c

.
Prof. Corjos Mirela Tatiana coala Gimnazial Nr. 25 Galai
Prof. oav Gabriela coala Gimnazial Nr. 18 Galai/ coala Sfntul Grigorie Teologul Galai
8. Artai c numrul

este divizibil cu 23.


Prof. Corjos Mirela Tatiana coala Gimnazial Nr. 25 Galai
Prof. oav Gabriela coala Gimnazial Nr. 18 Galai/ coala Sfntul Grigorie Teologul Galai
9. Aflai cel mai mic i cel mai mare numr de forma
divizibil cu 15.
Prof. Corjos Mirela Tatiana coala Gimnazial Nr. 25 Galai
Prof. oav Gabriela coala Gimnazial Nr. 18 Galai/ coala Sfntul Grigorie Teologul Galai
10. Aflai numerele naturale a i b astfel nct (a;b) = 15 i [a;b] = 90.
Prof. Drghici Ana Maria, Liceul "Constantin Brncoveanu" Horezu
11. Aflai numerele naturale a i b astfel nct (a;b) + [a;b] = 33.
Prof. Drghici Ana Maria, Liceul "Constantin Brncoveanu" Horezu
Rubric realizat de prof. Pit Rada-Marica , Drobeta Turnu Severin ,
Fleancu Mariana , Campulung Muscel
Setul II
12. Scriei numrul 18511 ca sum de dou ptrate perfecte.
Prof. Drghici Ana Maria, Liceul "Constantin Brncoveanu" Horezu
13. Determinai a,b,c N care verific relaiile:
3a+4b+5c=239 i 6b+5c=307,iar a este numr prim.
Prof. Ionic Constantin, Dr.Tr.Severin

21 ______________________________________________________________________________ Probleme propuse

14. S se determine toate numerele naturale nenule, care mprite la 29 dau ctul egal cu restul i
mprite la 35, dau ctul egal cu restul.
Prof. Ionic Constantin, Dr.Tr.Severin
15. Artai c un numr natural cu
cifre din care jumtate sunt cifra i jumtate sunt cifra nu
poate fi ptrat perfect.
Daniel Sitaru,Drobeta Turnu-Severin
16. Aflaicifrele a, b i c astfel nct:
abc 21 + 1abc = 15321 + abc
Prof. Drghici Ana Maria, Liceul "Constantin Brncoveanu" Horezu
17. Scriei numrul 18511 ca sum de dou ptrate perfecte.
Prof. Drghici Ana Maria, Liceul "Constantin Brncoveanu" Horezu
18. Cte numere naturale de cel mult 3 cifre mprite la 25 dau restul 10?
Prof. Fleancu Mariana, c. Gim.Nanu Muscel, Cmpulung Muscel, Arge
19. Aflai mulimile A si B dac ndeplinesc simultan condiiile: AUB={a,b,c}, A\B={a}
Prof. Fleancu Mariana, c. Gim.Nanu Muscel, Cmpulung Muscel, Arge
20. Calculati: 2
-(62
-2
)
Prof. Fleancu Mariana, c. Gim.Nanu Muscel, Cmpulung Muscel, Arge
21. Determinai numerele ab tiind c ab 2 = .....ab .
Prof. Fleancu Mariana, c. Gim.Nanu Muscel, Cmpulung Muscel, Arge
22. Fie n numrul numerelor naturale a cu proprietatea c 9
9 . Artai c n este cub perfect.
Prof. Fleancu Mariana, c. Gim.Nanu Muscel, Cmpulung Muscel, Arge
23. Aflai ctul i restul mpririi numrului 7 2
la5 2
.
Prof. Fleancu Mariana, c. Gim.Nanu Muscel, Cmpulung Muscel, Arge
24. Aflai
, astfel incat 3
4 se divide cu 5.
Prof. Fleancu Mariana, c. Gim.Nanu Muscel, Cmpulung Muscel, Arge
25. Scriei numrul 99
ca o sum de 3 cuburi perfecte.
Prof. Fleancu Mariana, c. Gim.Nanu Muscel, Cmpulung Muscel, Arge
26. Aflai exponentul lui 5 n descompunerea n factori primi a numrului a= 1x2x3x.......x2009
Prof. Fleancu Mariana, c. Gim.Nanu Muscel, Cmpulung Muscel, Arge
27. Fie N= paxqb , unde p,q sunt numere prime iar a,b N*. Determinai n dac numrul divizorilor lui n
este 9 iar suma divizorilor este 91.
Prof. Fleancu Mariana, c. Gim.Nanu Muscel, Cmpulung Muscel, Arge
Rubric realizat de prof. Livia Mocanu, Timisoara i
prof. Gabriela Soava, Galati
CLASA A VI-A

Setul I
1. Msurile a dou unghiuri complementare sunt direct proporionale cu numerele 7 i 8. Determinai
aceste msuri.
Prof. Corjos Mirela Tatiana coala Gimnazial Nr. 25 Galai
Prof. oav Gabriela coala Gimnazial Nr. 18 Galai/ coala Sfntul Grigorie Teologul Galai

5
2. Fie numerele a = i
6

. Determinai media aritmetic a numerelor a i b.

Prof. Corjos Mirela Tatiana coala Gimnazial Nr. 25 Galai


Prof. oav Gabriela coala Gimnazial Nr. 18 Galai/ coala Sfntul Grigorie Teologul Galai

Probleme propuse_______________________________________________________________________________________ 22

5
16
+ 0, (3) = .
6
3
Prof. Corjos Mirela Tatiana coala Gimnazial Nr. 25 Galai
Prof. oav Gabriela coala Gimnazial Nr. 18 Galai/ coala Sfntul Grigorie Teologul Galai
4. Un elev a cheltuit n patru zile o sum de bani, astfel: n prima zi 20% din sum, a doua zi o ptrime
din rest, a treia zi o treime din noul rest, iar n a patra zi 100 lei. Ce sum de bani a avut iniial elevul?
Prof. Corjos Mirela Tatiana coala Gimnazial Nr. 25 Galai
Prof. oav Gabriela coala Gimnazial Nr. 18 Galai/ coala Sfntul Grigorie Teologul Galai
5. Nou unghiuri n jurul unui punct au msurile exprimate prin numere naturale consecutive. Determinai
msurile unghiurilor.
Prof. Corjos Mirela Tatiana coala Gimnazial Nr. 25 Galai
Prof. oav Gabriela coala Gimnazial Nr. 18 Galai/ coala Sfntul Grigorie Teologul Galai
6. Un cltor pornete din localitatea A n localitatea B, aflat la distana de 74 km; din B pornete ctre
localitatea C, situat la 123 km de localitatea A, apoi mai departe ctre localitatea D, aflat la 141 km
de A. tiind c toate cele 4 localiti sunt aezate n linie dreapt, aflai la ce distan se afl
localitatea C fa de localitile B, respectiv D ?
Prof. Corjos Mirela Tatiana coala Gimnazial Nr. 25 Galai
Prof. oav Gabriela coala Gimnazial Nr. 18 Galai/ coala Sfntul Grigorie Teologul Galai
7. Un turist se deplaseaz ntre dou localiti cu o vitez medie de 5km/ or. n cte minute va strbate
distana de 1500m?
Prof. Corjos Mirela Tatiana coala Gimnazial Nr. 25 Galai
Prof. oav Gabriela coala Gimnazial Nr. 18 Galai/ coala Sfntul Grigorie Teologul Galai
8. n biblioteca unei coli a rmas, dup distribuirea ctre elevi, un numr de manuale de matematic.
Dac dintre acestea s-ar mai lua 12 manuale, ar rmne jumtate din numrul iniial al manualelor
rmase. Cte manuale rmseser?
Prof. Corjos Mirela Tatiana coala Gimnazial Nr. 25 Galai
Prof. oav Gabriela coala Gimnazial Nr. 18 Galai/ coala Sfntul Grigorie Teologul Galai
9. Scara unei hri este 1:10000. La ce distan se afl, una fa de cealalt, dou localiti A i B pe
hart, dac n realitate distana dintre ele este 3,45 km?
Prof. Corjos Mirela Tatiana coala Gimnazial Nr. 25 Galai
Prof. oav Gabriela coala Gimnazial Nr. 18 Galai/ coala Sfntul Grigorie Teologul Galai
10. Procentul de promovabilitate la matematic a elevilor claselor a-VI-a este, pe semestrul I, 98%. tiind
c doar un elev a rmas corigent, ci elevi sunt n clasele a-VI-a din acea coal?
Prof. Corjos Mirela Tatiana coala Gimnazial Nr. 25 Galai
Prof. oav Gabriela coala Gimnazial Nr. 18 Galai/ coala Sfntul Grigorie Teologul Galai
11. S se calculeze msurile a dou unghiuri suplementare , tiind c acestea sunt direct proporionale
cu msurile complementului si respective suplementului lor.
Prof. Ionic Constantin
12. S se determine a,b,c N* tiind c
= =
i 2a+3b+4c=87.
Prof. Ionic Constantin
13. Calculati unghiurile unui triunghi isoscel stiind ca o inaltime are lungimea jumatate din latura
corespunzatoare.
Prof. Hinoveanu Sorinel Vasile
14. Din cauza cresterii pretului materiilor prime, pretul unui produs a fost majorat cu 25%. Dupa numai o
luna, in urma aplicarii unei inovatii, noul pret a fost diminuat cu 40%. Cat a fost pretul initial al
produsului stiind ca ultimul pret al produsului este de 2016 lei.
Prof. Elena Rmnicianu
15. In exteriorul triunghiului dreptunghic isoscel ABC ([AB] [AC]) se construiesc triughiurile dreptunghice
isoscele ABD si ACE ( m(<D)=900 si m(<E)=900).
a. Sa se demonstreze ca punctele D, A, E sunt coliniare si ca DE // BC ;
b. Daca AB EC ={M} sa se arate ca AE este linie mijlocie in MCB.
Prof. Elena Rmnicianu
3. Rezolvai n Q ecuaia x =

23 ______________________________________________________________________________ Probleme propuse

16. Fie ABCD un patrulater convex astfel nct AC

BD =

. tiind c M, N, P, Q sunt mijloacele

laturilor AB, BC, CD, DA i c m(


,
i m(
cu numerele 0,2; 0,5 i 0,(3), studiai natura patrulaterului MNPQ.
17. Se consider patrulaterul convex ABCD cu M
i

. tiind c

(AD), N

sunt invers proporionale


Prof. Octavia Codru, Timioara

(DB) i P

(BC) astfel nct

, demonstrai c ABCD este paralelogram.


Prof. Octavia Codru, Timioara

18. Rezolvai ecuaia

, k N*.
Prof. Octavia Codru, Timioara

19. Rezolvai n Q ecuaia: 3(9 2 4 x 3 ) + 11 = 35 .

Prof. Veronica Riza, Timioara


+
+......+
Prof. Fleancu Mariana, c. Gim.Nanu Muscel, Cmpulung Muscel, Arge
21. Pe o dreapt d se iau punctele A,B,C,D astfel nct AD=x, BC=y, AB=z, AC=y-z, BD=x-z
a)Stabilii ordinea punctelor pe dreapt
b) Dac A este mijlocul segmentului CD i B este mijlocul segmentului AD, artai c
Prof. Fleancu Mariana, c. Gim.Nanu Muscel, Cmpulung Muscel, Arge
20. Calculai N=

Rubric realizat de prof. Draghici Ana Maria , Horezu , Valcea


Radulescu Valentin , Scornicesti ,Mitran Raluca , craiova
Setul II
22. Fie punctele coliniare
n, tiind c

, astfel nct

Determinai

Prof. Corjos Mirela Tatiana coala Gimnazial Nr. 25 Galai


Prof. oav Gabriela coala Gimnazial Nr. 18 Galai/ coala Sfntul Grigorie Teologul Galai
23. Determinai numerele x, y, z, tiind c x+y, y+z i x+z sunt direct proporionale cu 5, 9 i 12 i
.
Prof. Corjos Mirela Tatiana coala Gimnazial Nr. 25 Galai
Prof. oav Gabriela coala Gimnazial Nr. 18 Galai/ coala Sfntul Grigorie Teologul Galai
24. Rezolvai n N ecuaia ,
,
,
.
Prof. Corjos Mirela Tatiana coala Gimnazial Nr. 25 Galai
Prof. oav Gabriela coala Gimnazial Nr. 18 Galai/ coala Sfntul Grigorie Teologul Galai
25. Fie punctele A, B, C, D i O, astfel nct
,
i
. Demonstrai c DO AO.
Prof. Corjos Mirela Tatiana coala Gimnazial Nr. 25 Galai
Prof. oav Gabriela coala Gimnazial Nr. 18 Galai/ coala Sfntul Grigorie Teologul Galai
26. Artai c numrul a, care mprit la 65 d rest 39, este divizibil cu 13. Aflai astfel de numere ,
cuprinse ntre 200 i 300.
Prof. Corjos Mirela Tatiana coala Gimnazial Nr. 25 Galai
Prof. oav Gabriela coala Gimnazial Nr. 18 Galai/ coala Sfntul Grigorie Teologul Galai
27. Dac 2x+3y=11 i 2y+3z=17, artai c |
.
Prof. Corjos Mirela Tatiana coala Gimnazial Nr. 25 Galai
Prof. oav Gabriela coala Gimnazial Nr. 18 Galai/ coala Sfntul Grigorie Teologul Galai
28. Calculai:
.
Prof. Corjos Mirela Tatiana coala Gimnazial Nr. 25 Galai
Prof. oav Gabriela coala Gimnazial Nr. 18 Galai/ coala Sfntul Grigorie Teologul Galai

Probleme propuse_______________________________________________________________________________________ 24

29. Determinai a N astfel nct

Prof. Ionic Constantin


30. Determinati perechile de numere naturale (a,b) stiind ca indeplinesc simultan conditiile:
1) a+b=19
2) a este numar prim
3) b are exact 4 divizori
Prof. Aristita Pandioniu, Hinova
31. Sa se gaseasca numerele naturale prime a,b,c diferite intre ele, astfel ca suma S= +
sa fie
minima. Calculati aceasta suma.
Prof. Fluerau Anghel, Izvorul Brzii
32. Sa se determine toate fractiile de forma

care se pot simplifica cu 44.


Prof. Calafeteanu Gheorghe

12
6
33. Demonstrai c: 32 - 9 M 29.
Prof. Drghici Ana Maria, Liceul "Constantin Brncoveanu" Horezu
34. Se consider n unghiuri n jurul unui punct, n 2, exprimate prin numere naturale nenule. Fiecare
unghi este mai mare dect precedentul cu 60. Determinai numrul de unghiuri i msura fiecruia.
Prof. Drghici Ana Maria, Liceul "Constantin Brncoveanu" Horezu
35. Sa se afle x astfel incat 16 16 2 16 3 ... 16 2015 sa fie patratul numarului 16504x.
Prof. Elena Rmnicianu - Drobeta Turnu Severin
36. Fie
.
S se arate c suma cifrelor lui este divizibil cu .
Daniel Sitaru, Drobeta Turnu Severin
37. Aflai numerele prime a, b, c tiind c a + b + c = 1998 i b c = 42 .
Prof. Bloiu Daniela
38. S se determine toate numerele naturale mai mari decat 250 i mai mici dect 1200 care mprite la
numerele 28, 35, 40, 70 dau de fiecare dat restul 7.
Prof. Bloiu Daniela
39. Fie
32
63
3
224
3
672
2 2016 .S se arate c x
este divizibil cu2016 i c este ptrat perfect.
Prof. Elena Rmnicianu - Drobeta Turnu Severin
40. Demonstrati ca :
4
4
4
3
+
+ ... +
<
2009 3 7 2009 7 11
2009 2015 2019 2019 .
Prof. Elena Rmnicianu - Drobeta Turnu Severin
41. Sa se arate ca numerele 491008 + 7 2016 si 251008 + 2 5 2016 sunt prime intre ele.
Prof. Elena Rmnicianu - Drobeta Turnu Severin
1
1
1
2015
+
+
+
<
...
2
32
2016 2 2016
42. Artai c: 2
Prof. Elena Rmnicianu - Drobeta Turnu Severin
43. Artai c numrul n=1900+110111- 1102111+1103111-1104111++ 11011111-11012111+11013
este divizibil cu 67.
Prof. Elena Rmnicianu - Drobeta Turnu Severin
44. Sa se rezolve ecuatia :
x +1 x + 2
x + 2015
+
+ ... +
= 2015
2
3
2016
.
Prof. Elena Rmnicianu - Drobeta Turnu Severin

45. Scrieti in ordine crescatoare fractiile:

25 ______________________________________________________________________________ Probleme propuse

n + 2015
n + 2012
n + 2013 n + 2016
;
;
n + 2013
n + 2014 n + 2015 i n + 2014 .

Prof. Elena Rmnicianu - Drobeta Turnu Severin


46. Aflai numerele naturale a i b astfel nct (a;b) = 15 i [a;b] = 90.
Prof. Drghici Ana Maria, Liceul "Constantin Brncoveanu" Horezu
47. Aflai numerele naturale a i b astfel nct (a;b) + [a;b] = 33.
Prof. Drghici Ana Maria, Liceul "Constantin Brncoveanu" Horezu
12
6
48. Demonstrai c: 32 - 9 M 29.
Prof. Drghici Ana Maria, Liceul "Constantin Brncoveanu" Horezu
49. Se considera n unghiuri n jurul unui punct, n 2, exprimate prin numere natural nenule.
Fiecare unghi este mai mare dect precedentul cu 60.
Determinai numrul de unghiuri i msura fiecruia.
Prof. Drghici Ana Maria, Liceul "Constantin Brncoveanu" Horezu
50. Aflai restul mpririi numrului 5a+22 (unde a N ) la 19, tiind c 19|2a+57.
Prof. Fleancu Mariana, c. Gim.Nanu Muscel, Cmpulung Muscel, Arge
51. Aflai numerele naturale x i y tiind c [x,y]=15(x,y) i xy=135.
Prof. Fleancu Mariana, c. Gim.Nanu Muscel, Cmpulung Muscel, Arge
52. Ci termeni trebuie adunai la numrul 1+2+3+ pentru a obtine un rezultat de forma xxx ?
Prof. Fleancu Mariana, c. Gim.Nanu Muscel, Cmpulung Muscel, Arge
53. Se dau semidreptele opuse[OA si [OB. n acelai semiplan determinat de dreapta AB se iau punctele
C,D,E i F astfel nct unghiurile AOC, COD, DOE, EOF i FOB sunt adiacente dou cte dou i
m(<COD)=m(<AOC) + 1 , m(<DOE)=m(<COD) + 1 ,m(<EOF)=m(<DOE) + 1 i m(<FOB)=m(<EOF)
+ 1. Aflai msurile acestor unghiuri.
Prof. Fleancu Mariana, c. Gim.Nanu Muscel, Cmpulung Muscel, Arge
Rubric realizat de prof. Constantin Bolbotin, Bile Herculane ,
prof. Rmniceanu Elena, Dr.Tr. Severin, prof. Dana Cotfasa, Craiova

CLASA A VII-A

Setul I
1. S se arate c oricare ar fi a,b,cZ care verific relaia 2a+3b=5c, atunci

(b-a)(c-b)(a-c) este divizibil cu 30.

Prof. Ionic Constantin, Drobeta Turnu Severin


2. Dac a,b,c R astfel nct a+2b+3c=7 ,artai c
(a- +(b-2 +(c-3 = + + .
Prof. Ionic Constantin, Drobeta Turnu Severin
3. Rezolvati ecuatia:
1
1
Prof. Elian Neamu, Bistrita
4. Demonstrai c numrul N =

26 43

302

este

ptrat perfect.
Prof. Octavia Codru, Timioara
5. n ABC, M este mijlocul lui [BC], N este mijlocul lui [AM], iar NP || BC cu P (AB). Dac aria ABC
este de 104 cm 2 , aflai aria BPN.
Prof. Drghici Ana Maria, Liceul "Constantin Brncoveanu" Horezu

Probleme propuse_______________________________________________________________________________________ 26

6. Fie ABCD un paralelogram, AD=2 AB, m( A) =60o. Se considera punctele M (AD), N(BC) astfel
incat AM= AD, CN= BC. Artai ca patrulaterul MBND este dreptunghi.
Prof. Chievescu Tatiana, Liceul Constantin Brncoveanu Horezu Vlcea
7. Fie ABC si punctele D si E simetricele punctelor B, C fata de A.
a) artai ca patrulaterul BCDE este paralelogram
b) stabilii natura ABC, astfel incat BCDE sa fie ptrat.
Prof. Chievescu Tatiana, Liceul Constantin Brncoveanu Horezu Vlcea
8. Artai ca
< 2.
Prof. Chievescu Tatiana, Liceul Constantin Brncoveanu Horezu Vlcea
9. Calculai :

Prof. Chievescu Tatiana, Liceul Constantin Brncoveanu Horezu Vlcea


10. S se arate c oricare ar fi a,b,cZ care verific relaia 2a+3b=5c, atunci
(b-a)(c-b)(a-c) este divizibil cu 30.
Prof. Ionic Constantin, Drobeta Turnu Severin
11. Dac x,yR* astfel nct :

-2x7-2y6+13=0,s se arate c

)(x-y)=1

Prof. Ionic Constantin, Drobeta Turnu Severin


12. Dac x9, y16 i z25,artai c are loc relaia
9+4

16 + 5

25

n ce caz are loc egalitatea?


13. Fie

. S se arate c

Prof. Ionic Constantin, Drobeta Turnu Severin


nu este numr natural.
Daniel Sitaru, Drobeta Turnu-Severin

14. Aratati ca numarul :

131313132 12121212 2
este numar rational.
7070707 2 6060606 2 2020202 2
Prof. Elena Rmnicianu - Drobeta Turnu Severin
15. In paralelogramul ABCD, msura unghiului B este egal cu suplementul unghiului de 600, AB = 12
cm, iar BC este egal cu doua treimi din AC. Dac M este mijlocul laturii AB, s se afle DM, perimetrul
i aria paralelogramului.
Prof. Elena Rmnicianu - Drobeta Turnu Severin
Rubric realizat de prof. Mariana Draga Tatucu , Dr.Tr.Severin , Tudosie Gina ,
Strehaia , Tudosie Mircea , Strehaia

Setul II
16. n
,
2
. Dac

90,
18 ,

astfel nct
3 ,
astfel nct
12 artai c
.
Prof. Draga Ttucu Mariana, Colegiul Naional Gheorghe ieica
17. S se arate c nu exist triunghi care are lungimile nlimilor egale cu 1, 5 respectiv, 5 2 .
Prof. Draga Ttucu Mariana, Colegiul Naional Gheorghe ieica
|6
| 2 13 3
18. Aflai perechile naturale ,
astfel nct
.
Prof. Draga Ttucu Mariana, Colegiul Naional Gheorghe ieica
19. n
, avem
,
i
astfel nct

27 ______________________________________________________________________________ Probleme propuse

Dac

, artai c

20. Sa se rezolve ecuatia in

este median n
.
Prof. Draga Ttucu Mariana, Colegiul Naional Gheorghe ieica

:
,

unde
respectiv
pozitiv .

, reprezinta partea fractionara respective partea intreaga a numarului rational

Studenta Neciu Denisa Nicoleta, Facultatea de Matematica si Informatica,


Universitatea din Bucuresti
21. Determinati numerele natural nenule , , astfel incat :

Studenta Neciu Denisa Nicoleta, Facultatea de Matematica si Informatica,


Universitatea din Bucuresti
22. Sa se rezolve in X ecuatia :
Studenta Neciu Denisa Nicoleta, Facultatea de Matematica si Informatica,
Universitatea din Bucuresti
2015
2015
2
2
a
,
b

Z
(
a
23. tiind c
i a 2ab + 2b 2a + 2 = 0 , s se determine 3) + (b 1) .
Prof. Veronica Riza, Timioara
24. Dac a = 21 8 5 17 + 12 2 i b = 12 8 2 + 5 , s se calculeze (a + b ) .
Prof. Veronica Riza, Timioara
25. Se consider rombul ABCD cu m( A) = 60 o i AC BD = {O}. Paralela prin C la BD
intersecteaz AD n E i F este simetricul punctului O fa de punctul D .
S se arate c: a) ABCE este trapez isoscel.
b) OCEF este dreptunghi.
c) ABCD = COFE .
Prof. Veronica Riza, Timioara
26. n ptratul ABCD se consider: M i Q mijloacele laturilor ( AD ) , respectiv (DC ) , punctul
N (BC ) astfel nct BN = 2 NC i {P} = MN DC , {R} = NQ AD . S se arate c
NP MR DQ

= 1.
MN DR PQ
Prof. Veronica Riza, Timioara
27. n ABC, M este mijlocul lui [BC], N este mijlocullui [AM], iar NP || BC cu P (AB). Dac aria ABC
este de 104 cm 2 , aflai aria BPN.
Prof. Drghici Ana Maria, Liceul "Constantin Brncoveanu" Horezu
28. Fie ABCD un paralelogram, AD=2 AB, m( A) =60o. Se considera punctele M (AD), N(BC) astfel
incat AM= AD, CN= BC. Artai ca patrulaterul MBND este dreptunghi.
Prof. Chievescu TatianaLiceul Constantin Brncoveanu Horezu Vlcea
29. Fie ABC si punctele D si E simetricele punctelor B, C fata de A.
a) artai ca patrulaterul BCDE este paralelogram
b) stabilii natura ABC, astfel incat BCDE sa fie ptrat.
Prof. Chievescu Tatiana Liceul Constantin Brncoveanu Horezu Vlcea
2015

30. Calculai :

Prof. Chievescu Tatiana Liceul Constantin Brncoveanu Horezu Vlcea


Rubric realizat de prof. Riza Veronica , Timisoara i
prof. Nicolae Elena,Craiova, prof. Adriana Maris , Lugoj

Probleme propuse_______________________________________________________________________________________ 28

CLASA A VIII-A

Setul I
1. Artai c numrul 3

.... 3
este iraional.
Prof. Ionic Constantin, Drobeta Turnu Severin

2. Se d x+ =5, x R\{0}. Calculati:


a)

, c)

, b)

Prof. Mari Marinela

3. Aratati ca expresia E(x)=

este independenta de x.
Prof. Calafeteanu Gheorghe

4. Rezolvati ecuatia (5-m)(4-m)x=5-m, m R,x

Prof. Calafeteanu Gheorghe

5. Rezolvati ecuatia , stabilind si domeniul maxim de definitie :


3
2
1
2
5
6

Prof. Mari Marinela


6. Se da tetraedrul ABCD. Stiind ca AB=20cm, CD=30cm si AB CD, E (AB), F (CD), EF AB, EF
CD, EF=10cm, aflati volumul piramidei ABCD.
Prof. Calafeteanu Gheorghe
7. Sa se calculeze volumul unui paralelipiped dreptunghic a carui arie totala este egala cu 248
, iar
dimensiunile sunt direct proportionale cu 2, 3 si 5.
Prof. Vasilcan Tiberiu, Orsova
8. Fie , ,
. S se arate c:
Daniel Sitaru, Drobeta Turnu-Severin
9. Rezolvai n Q ecuaia:
2

x4 +

1 2 1
+x 2 = 2
x4
x
.

Prof. Drghici Ana Maria, Liceul "Constantin Brncoveanu" Horezu


10. Determinai x,y,z R astfel nct x+y+z=12 i xy+xz+yz=48
Prof. Ionic Constantin, Drobeta Turnu Severin
11. S se rezolve ecuaiile i raionale n R
a)7 3 + 6 3 =5
b)4
1 - 6
= 2
3
Prof. Ionic Constantin, Drobeta Turnu Severin
12. S se arate c oricare ar fi x R+ are loc relaia:3 +4
2014+5
20156x+2016
Prof. Ionic Constantin, Drobeta Turnu Severin
Rubric realizat de prof. Calafeteanu Gheorghe,
Prof. Maris Marilena, Dr. Tr. Severin

29 ______________________________________________________________________________ Probleme propuse

Setul II
13. Fie a,b,c Q+ astfel nct

Artai c

14. Fie
i

Prof. Ionic Constantin, Drobeta Turnu Severin


un tetraedru cu
. Dac
;
;
nlimea din a tetraedrului s se arate c:

Daniel Sitaru, Drobeta Turnu Severin


0, pentru orice numar real x.
Prof. Calafeteanu Gheorghe
+6
11
6
4 este divizibila cu 4 pentru orice

15. Sa se arate ca

16. Sa se demonstreze ca expresia E(a)=


valoare, numar intreg, a lui a.

Prof. Calafeteanu Gheorghe


5

17. Rezolvati in R ecuatia:

=2
Prof. Calafeteanu Gheorghe

18. Sa se rezolve in multimea numerelor intregi ecuatia:


2
19. Se da fractia: F(x)=
F(x) este un numar prim.

Prof. Calafeteanu Gheorghe

. Sa se simplifice fractia si sa se determine x

pentru care

Prof. Mari Marinela

20. Se da expresia E(x)=


. Sa se determine valoarea maxima a lui E(x), x
si valoarea lui x
pentru care se obtine E(x) maxim.
Prof. Calafeteanu Gheorghe
21. Stabiliti daca numarul
4
4
4
4
4
+
+
+ +
+
a=
504 508 508 512 512 516
2008 2012 2012 2016
1
1
apartine intervalului (
).
;
1008 504
Prof. Elena Rmnicianu - Drobeta Turnu Severin
22. Rezolvai n Q ecuaia:
2

x4 +

1 2 1
+x 2 = 2.
x4
x
Prof. Drghici Ana Maria, Liceul "Constantin Brncoveanu" Horezu

Rubric realizat de prof. Diaconescu Manuela, Braila,


prof. Alin Bacila, Motru ,prof.dr. Bogdan Heroiu , Campulung, Arges

Probleme propuse_______________________________________________________________________________________ 30

CLASA A IX-A
Setul I
,

1. Artai c dac

,,
atunci

.
Daniel Sitaru, Drobeta Turnu Severin

2. Fie , ,

, . S se arate c:

Daniel Sitaru, Drobeta Turnu Severin


3. Sa se calculeze :
a) :
b)
c)
d)

Prof. Radulescu Valentin, Scornicesti


4. Fie numerele reale strict pozitive x, y i z, astfel nct 2 x + 3 y + 4 z = 6 . S se arate c
9
2x + 3y + 4z < .
2
Prof. dr. Mihaela Albici, coala Gimnazial Muereasca de Sus, Jud. Vlcea
5. S se determine numerele reale x, y i z care verific egalitatea:

3 2 y + z 2 + 2 10 y z x y z + 2 + 4 x + z = 14 + z + y x 2
Prof. dr. Mihaela Albici, coala Gimnazial Muereasca de Sus, Jud. Vlcea
6. S se calculeze suma:

S n = 1 + 12 + 112 + ... + 111


12...
312 ,pentru n 2 .
( n 1)ori

Prof. dr. Mihaela Albici, coala Gimnazial Muereasca de Sus, Jud. Vlcea
7. Artai ca
< 2.
Prof. Chievescu Tatiana Liceul Constantin Brncoveanu Horezu Vlcea
8. Dac
este a n-a zecimal a numrului ,
este a n-a zecimal a numrului i este a n-a
zecimal a numrului

, s se arate c numrul
este divizibil cu 7.
Prof. Barbu Daniela, Climneti, Prof. Neacu Stelua, Climneti
9. S se rezolve ecuaia 2
6
5
12
20 0.
Prof. Barbu Daniela, Climneti, Prof. Neacu Stelua, Climneti
10. S se rezolve ecuaia
.
Prof. Barbu Daniela, Climneti, Prof. Neacu Stelua, Climneti
uuuur 1 uuuur r uuur
r uuuur
11. In paralelogramul ABCD se considera: a = AM , unde M BC , BM = MC si b = AN , unde
2
uuur 1 uuur
uuur uuur uuur uuuur uuuur uuur
r r
N DC , DN = NC . Calculati, in functie de a si b , vectorii: AB, AD, BD, MNsiCM + ND
2
Prof. Barbu Daniela, Climneti, Prof. Neacu Stelua, Climneti
12. Se d triunghiul
i punctele
i
astfel nct
i
.
a) Determinai
astfel nct dreptele
,
,
s fie concurente ntr-un punct T.

31 ______________________________________________________________________________ Probleme propuse

i
.
Prof. Barbu Daniela, Climneti, Prof. Neacu Stelua, Climneti
3n + 2
13. Se d irul (an ) n1 cu termenul general an =
.
2n 1
a) Studiai monotonia i mrginirea irului;
b) Determinai rangul termenului pentru care 11an +1 9an = 2
Prof. Barbu Daniela, Climneti, Prof. Neacu Stelua, Climneti
14. Se d progresia aritmetic (an ) n1 cu termenii numere reale pozitive si de raie r .
b) S se exprime

Notm

n funcie de

a1 a3 + a2 a4 + ... + an 2 an = T .

S se arate c:

1
a1 + an
a1 + an
1
1
( n 2 ) r 2 + + ... +
( n 2)
T
2
2
an 1
a2 a3
Prof. Barbu Daniela, Climneti, Prof. Neacu Stelua, Climneti

Setul II
15. Rezolvai n N* ecuaia:
[ 1 2 ] + [ 2 3 ] + + [ n (n + 1) ] = 45 , unde [x] este partea ntreag a lui x.
Prof. Drghici Ana Maria, Liceul "Constantin Brncoveanu" Horezu
16. Se consider funcia f : R R, f ( x ) = 3x 1.

a) S se arate c numerele f (2 ), f (4 ) i f (6 ) sunt n progresie aritmetic.


b) S se calculeze S = f (0) + f (1) + f (2) + ... + f (100) .
Prof. dr. Mihaela Albici, coala Gimnazial Muereasca de Sus, Jud. Vlcea
17. Calculai :

Prof. Chievescu Tatiana Liceul Constantin Brncoveanu Horezu Vlcea


18. Rezolvai n N* ecuaia:
[ 1 2 ] + [ 2 3 ] + + [ n (n + 1) ] = 45, unde [x] este partea ntreag a lui x.
Prof. Drghici Ana Maria, Liceul "Constantin Brncoveanu" Horezu
19. Rezolvai n Q ecuaia:
2

x4 +

1 2 1
+x 2 = 2.
x4
x
Prof. Drghici Ana Maria, Liceul "Constantin Brncoveanu" Horezu

20. S se arate c numrul


98 406
numr natural.
21. S se arate c
a)
b)
c)

380

12010

405

1805

99

702

73

403 este

Prof. Barbu Daniela, Climneti, Prof. Neacu Stelua, Climneti

,
,

,
.

,
Prof. Barbu Daniela, Climneti, Prof. Neacu Stelua, Climneti
|.
22. Se d funcia :
,
3
2|3
3
a) Determinai punctele de intersecie ale graficului funciei cu axele de coordonate.
b) Calculai numerele

Probleme propuse_______________________________________________________________________________________ 32

, unde

prin
s-a notat partea ntreag a numrului .
i verificai dac punctul
,
aparine graficului funciei .
Prof. Barbu Daniela, Climneti, Prof. Neacu Stelua, Climneti
x + 1, x 0
3, x 1

23. Se considera functiile f , g : , f ( x) = - x + 1, x (0;3) si g ( x) =


.
2 x + a, x > 1

- 2, x 3
a) Aflati valorile lui
astfel incat functiile f o g si g o f sa fie constante.

b) Pentru a=1 calculati si demonstrati prin inductie matematica suma:


1
1
1
S=
+
+ ..... +
g (1) g (2) g (2) g (3)
g (n) g (n + 1)
Prof. Barbu Daniela, Climneti, Prof. Neacu Stelua, Climneti
Rubric realizat de prof. Barbu Daniela, Climneti i
prof. Neacu Stelua, Climneti

CLASA A X-A
Setul I
1. Fie

,,

2. S se rezolve n
3. Fie

;
. Dac |

ecuaia:

,,

; ,
| |;

, ;
;
i
, s se arate c
.
Daniel Sitaru, Drobeta Turnu Severin
|

|
Daniel Sitaru, Drobeta Turnu Severin

S se arate c:

Daniel Sitaru, Drobeta Turnu Severin


4. S se arate c funcia f : R R , f ( x) = x 2 2015 x + 2014 nu este injectiv.
Prof. Ovidiu Ticui, Dr. Tr. Severin
x + x 2 + 27 + 3 x x 2 + 27 = 2
Prof. Doru Preneanu, Dr. Tr. Severin
6. Fie funcia f : R R, avnd proprietatea c f ( f ( x)) = 2015 x 2014, x R.
S se arate c funcia f este o funcie inversabil.
Prof. Ovidiu Ticui, Dr. Tr. Severin
x 3 + ( y 3)i
7. S se determine numerele reale x,y tiind c:
=i
x + 2 + ( y + 4 )i
Prof. Doru Preneanu, Dr. Tr. Severin

5. S se rezolve ecuatia:

(
2015

8. S se verfice dac fracia

2015

k =2

2016

2015

1
k +1

este supraunitar.
Prof. Ovidiu Ticui,Dr.Tr.Severin

33 ______________________________________________________________________________ Probleme propuse

9. Fie

z=

cos 2015 + i sin 2015


sin 2016 + i cos 2016 i = z 2015 . Aflai | | .

Prof. Ovidiu Ticui,Dr.Tr.Severin


10. S se verfice dac numrul = 1 + lg 2015 lg 20150 lg 201500 lg 2015000 se poate scrie ca
ptratul unui numr real.
Prof. Ovidiu Ticui,Dr.Tr.Severin
11. S se determine funcia exponenial, strict cresctoare f : R (0, ) , care are proprietatea:
10
f (1) + f (1) =
.
3
Prof. Ovidiu Ticui, Dr. Tr. Severin
12. S se rezolve ecuaia:

5 5 x = x

Prof. Doru Preneanu, Dr. Tr. Severin


a k + ibk

13. S se calculeze
, unde a k , bk R , k {1,2,...,2015}.
k =1 bk ia k
Prof. Ovidiu Ticui, Dr.Tr.Severin
14. Fie z C astfel nct z 1 . S se arate c este adevrat inegalitatea:
2015

(1 + i ) 2014 z + 2 2014 < 2 2015 .


m +1
15. Determinai m real astfel nct z =
R
m 1 + mi

Prof. Ovidiu Ticui, ,Dr.Tr.Severin

Prof. Doru Preneanu, Dr. Tr. Severin


16. S se determine m real tiind c inegalitatea log m 1 ( x 2 + 2010) 1 este adevarat pentru orice x
m +1

real.

Prof. Doru Preneanu, Dr. Tr. Severin

Setul II
17. S se arate c funcia f : R R , f ( x) = 2015 x + 2014 este bijectiv.
Prof. Ovidiu Ticui, Dr. Tr. Severin
1
1
18. Fie funcia f : C C , f ( z ) = . Dac | |= 1 , s se arate c f ( ) =
.
z
f ( )
Prof. Ovidiu Ticui, Dr. Tr. Severin
x +1
1
19. S se rezolve n R ecuaia 9 2 = x .
3
Prof. Ovidiu Ticui, Dr. Tr. Severin
20. S se rezolve pe (1, ) ecuaia lg( x + 1) + lg( x 1) = 2 lg 3 + lg 11 .
Prof. Ovidiu Ticui, Dr. Tr. Severin
x 3 + ( y 3)i
=i
x + 2 + ( y + 4 )i
Prof. Doru Preneanu, Dr. Tr. Severin
Dac z1 z 2 sunt numere complexe, s se arate c z1 z 2 z1 z 2 , este pur imaginar
Prof. Doru Preneanu, Dr. Tr. Severin
Determinai valorile lui x pentru care este definit expresia: x 2 + log 2 (3x-5).
Prof. Doru Preneanu, Dr. Tr. Severin
Determinai numerele complexe z care verific relaia: z 2 = 3 + 4i
Prof. Doru Preneanu, Dr. Tr. Severin
Aduceti urmtoarea expresie la forma cea mai simpl:

21. S se determine numerele reale x,y tiind c :


22.
23.
24.
25.

Probleme propuse_______________________________________________________________________________________ 34
3

E=

a2 b2
1
2

a +b

1
2

a + a 2b 2 + b
:
+ 2a 2 b 2
ab

Prof. Doru Preneanu, Dr. Tr. Severin


26. S se rezolve urmtoarele inecuaii :
a)
;
b)
0;

c)
d)

3;

e)

.
Prof. Doru Preneanu, Dr. Tr. Severin
Rubric realizat de prof.Ovidiu Ticui i
prof.Doru Preneanu,Dr.Tr.Severin

CLASA A XI-A
Setul I
1. Se consider funcia

f ( x) =

,tiind c exist un unic b

2. Dac

A, B M 2 (

4x2 1
, f : D .Determinai valorile parametrului real a
2x + a2 x a
pentru care lim f ( x) nu este finit.
2

x b
x <b

Prof. Dan Nedeianu, C.N. Gh. ieica

) , astfel nct ( 2 A I 2 ) B = 4 A , s se arate c det ( A2 + B 2 ) 0 .

Prof. Dan Nedeianu, C.N. Gh. ieica


3. Dac a, b, c , bc > a > 0 > b , s se arate c ecuaia ax 3 + bx 2 + cx + 1 = 0 are trei rdcini
reale.
Prof. Dan Nedeianu, C.N. Gh. ieica
4. Dac a , a > 0 , s se calculeze :

lim
x

2 x + 1 + 8 x + 2 ax + 3

) , discuie.

Prof. Dan Nedeianu, C.N. Gh. ieica


det ( A + B ) + det ( A B ) = 2 det A + 2 det B,
pentru
orice

5. a)
Demonstrai
identitatea
A, B M 2 ( )
.
2
b) Demonstrai c dac A M 2 ( ) atunci (1 + det A ) det ( A2 I 2 ) .

Prof. Dan Nedeianu, C.N. Gh. ieica


6. S se studieze convergena irului
;
; !
!

Daniel Sitaru, Drobeta Turnu Severin

7. S se rezolve in numere reale sistemul:

Daniel Sitaru, Drobeta Turnu Severin

35 ______________________________________________________________________________ Probleme propuse

8. S se calculeze:
!
!

9. Fie matricele

A, B M 3 (R ) , unde

Daniel Sitaru, Drobeta Turnu Severin


1 7 13

A = 0 1 19 i B = A I 3 . Demonstrai c
0 0 1

n(n 1) 2
B , pentru ()n 3 .
2
Prof. Amariei Laura-Georgeta, Colegiul Tehnic de Alimentaie i Turism
Dumitru Mooc Galai i prof. Amariei Lucian
cos x sin x
(
)
A

M
R
2
consider
matricea
A =
,
Demonstrai
c
.
sin x cos x

An = I 3 + nB +

10. Se

( )

Tr A3 = 2 cos x(2 cos 2 x 1) .

Prof. Amariei Laura-Georgeta, Colegiul Tehnic de Alimentaie i Turism


Dumitru Mooc Galai i prof. Amariei Lucian
8 11 14
2
11. S se scrie termenul general al irului 3 , 1 , 7 , 9 , 11 , ...
Prof. Amariei Laura-Georgeta, Colegiul Tehnic de Alimentaie i Turism
Dumitru Mooc Galai i prof. Amariei Lucian
12. Fie irul

()n 1 .

(xn )n1

cu x1 = 1 i xn +1 = xn + 6 n . Artai c x1 + x2 + K + xn = n3 , pentru
Prof. Amariei Laura-Georgeta, Colegiul Tehnic de Alimentaie i Turism
Dumitru Mooc Galai i prof. Amariei Lucian

13. Artai c irul

(xn )n1

cu termenul general xn = n 2 3n + 6 n 2 3n + 3 este un ir

mrginit.
Prof. Amariei Laura-Georgeta, Colegiul Tehnic de Alimentaie i Turism
Dumitru Mooc Galai i prof. Amariei Lucian
Setul II

14. Se consider funcia f :


3
2 2
a x + bx + 4 , x > 1

f ( x ) = a + b + c , x = 1

1
b 2 x3 + ax + , x < 1
4

15. Determinai a, b, c ,tiind c funcia f este continu n punctul x0 = 1 .


Prof. Dan Nedeianu, C.N. Gh. ieica
sin x )( tg 2 x )
(
lim
2
x0
( e3x 1)
16. Calculai
Prof. Dan Nedeianu, C.N. Gh. ieica
ln ( 3x 2 + 1)
lim
17. Calculai x0 cos 4 x 1 .
Prof. Dan Nedeianu, C.N. Gh. ieica

Probleme propuse_______________________________________________________________________________________ 36

1 2
A=
M2 ( )
2 3
18. Se consider matricea
.
a) Demonstrai c exist p, q astfel nct A1 = pA + qI 2 .
b) Exist matricea X M 2 (

) astfel nct

X X T = A?
Prof. Dan Nedeianu, C.N. Gh. ieica

1
a
9
19. Considerm determinantul D(a ) = 2 a + 2 20 a , a
a2
1 3

D(a ) = ( a 2 ) ( a + 3) , a .
2

a) Artai c

b) Determinai m , pentru care D (m) = 4 .


D ( 2 x 6 ) = 0, x .
c) Rezolvai ecuaia
.
Prof. Dan Nedeianu, C.N. Gh. ieica
2
x
20. Fie matricea A M 2 (R ) , unde A =
. S se rezolve ecuaia det A + Tr ( A) = 3 .
3 2 x 1
Prof. Amariei Laura-Georgeta, Colegiul Tehnic de Alimentaie i Turism
Dumitru Mooc Galai i prof. Amariei Lucian
1 0 a
i B M 3 x 2 (R ) ,
21. S se determine det ( A B ) , unde A M 2 x 3 (R ) , A =
a 0 1
1 1

B= 0
b .
1 1

Prof. Amariei Laura-Georgeta, Colegiul Tehnic de Alimentaie i Turism


Dumitru Mooc Galai i prof. Amariei Lucian
22. S se demonstreze c nu exist dou funcii f i g , astfel nct f , g : D R definite prin

f ( x ) = log 2 x 2 3 x + 2 i g ( x ) = (1 x )( x 2 ) .
Prof. Amariei Laura-Georgeta, Colegiul Tehnic de Alimentaie i Turism
Dumitru Mooc Galai i prof. Amariei Lucian
sin 3 x sin 5 x sin 7 x K sin 2015 x
<1
lim
23. Demonstrai c x 0 sin 2 x sin 4 x sin 6 x K sin 2014 x
.
Prof. Amariei Laura-Georgeta, Colegiul Tehnic de Alimentaie i Turism
Dumitru Mooc Galai i prof. Amariei Lucian
ln x + 3 x + a, x > 1
24. S se determine a R astfel nct funcia f : R R dat prin f (x ) =
s
x1
2 x + 5,
aib limit n punctul x0 = 1 .
Prof. Amariei Laura-Georgeta, Colegiul Tehnic de Alimentaie i Turism
Dumitru Mooc Galai i prof. Amariei Lucian

Rubric realizat de prof. Amariei Laura-Georgeta, Galai i


prof. Dan Nedeianu, Drobeta Turnu Severin

37 ______________________________________________________________________________ Probleme propuse

CLASA A XII-A
Setul I
1. Pe mulimea

definim legea de compoziie

2. S se calculeze:

Daniel Sitaru, DrobetaTurnu-Severin

3. S se calculeze:

4. Fie

:
,

o funcie continu n
0. S se arate dac
atunci:

Daniel Sitaru, DrobetaTurnu-Severin


,
, integrabil pe
i

Daniel Sitaru, DrobetaTurnu-Severin


5. S se calculeze:

Daniel Sitaru, Drobeta Turnu Severin


, un polinom avnd rdcinile:
, ,,

6. Fie
.
S se calculeze:

Daniel Sitaru, Drobeta Turnu Severin


7. Sa se calculeze
a)

b)
c)

d)
Prof. Radulescu Valentin
8. Fie legea de compozitie
a) Sa se calculeze
b) Sa se rezolve ecuatia
c) Sa se arate ca legea este comutativa.

, ,

Prof. Radulescu Valentin

9. Se dau integralele
a) Calculai ,
.
b) Calculai
.

Probleme propuse_______________________________________________________________________________________ 38

Artaic

Prof. Barbu Daniela, Climneti, Prof. Neacu Stelua, Climneti


xy
10. Pe Q se definete legea de compoziie x y = x + y
. Demonstrai c A=Q-{2011} este parte
2011
stabil fa de i c (A, ) este grup abelian; Calculai 1 2 3 ... 2011 .
Prof. Barbu Daniela, Climneti, Prof. Neacu Stelua, Climneti
Setul II

2 x + 15 3
, daca x < 3
3
x2 9

a x + 4 + b
11. Se d funcia f: R R, f(x)=
, daca x [3,5] . Determinai numerele reale a i b
x+4

e x 5 + x 6
+ 1 , daca x > 5

x 5
pentru care funcia admite primitive pe R; Pentru numerele a i b determinate la subpunctul
4

precedent, calculai ( x 5) f ( x)dx .


0

Prof. Barbu Daniela, Climneti, Prof. Neacu Stelua, Climneti


12. Pe R se consider legea de compoziie x * y = 2015 x + 2015 y + xy .
a) Verificai daca: x * y = (2015 + x)(2015 + y ) - 20152 .
b) Verificati asociativitatea legii date.
c) Rezolvai ecuaia x * x = (- 2015) *0 .
Gsii dou elemente a, b R\ Z pentru care a * b Z.
Prof. Barbu Daniela, Climneti, Prof. Neacu Stelua, Climneti
2012 x + 1006
13. Se consider funciile f,g:(0, ) R, f(x)=2012 x +503ln(x) i g(x)=
2x
a) Artai ca funcia f este o primitiv a funciei g;
b) S se arate c orice primitiv a funciei f este strict cresctoare pe [1, ];
c) Aflai o primitiv F a funciei f pentru care F(1)=
d) Sa se calculeze
e

f ( x) g ( x)dx
1

.
Prof. Barbu Daniela, Climneti, Prof. Neacu Stelua, Climneti

14. Pe multimea G=(1,+ ) se considera legea de compozitie x y =


a) Sa se arate ca ( G,) este grup abelian;

x2 y2 x2 y2 + 2 .

b) Determinati a,b R pentru care functia f:(0, ) R, f(x)= ax + b este izomorfism intre grupurile (
R+* , ) si ( G,)
15. Pentru n N se consider funciile fn: [-1,1] R, fn(x)=(x2+1)enx.
1

a) S se calculeze

f 3 ( x)
dx ;
2
+1

b) S se calculeze

Prof. Cotoarba Cristian, Rm. Valcea

( x)dx ; c) S se arate c 2 f 2010 ( x)dx


1

2008

( x)dx + f 2012 ( x)dx


1

39 ______________________________________________________________________________ Probleme propuse

Prof. Mtcu Dumitru, Rm. Vlcea


16. Pe mulimea numerelor reale se definete legea de compoziie dat prin x y = x + y + xy .
Demonstrai
c
legea
de
compoziie
este
asociativ
i
calculai
( 100 ) ( 99 ) K ( 1) 0 1 K 100 .
Prof. Amariei Laura-Georgeta, Colegiul Tehnic de Alimentaie i Turism
Dumitru Mooc Galai i prof. Amariei Lucian
17. Pe intervalul (0,+ ) se definete legea de compoziie dat prin x y = x ln y . Demonstra c legea
de compoziie este comutativ.
Prof. Amariei Laura-Georgeta, Colegiul Tehnic de Alimentaie i Turism
Dumitru Mooc Galai i prof. Amariei Lucian
x+ y
18. S se arate c legea de compoziie x y =
este bine definit pe mulimea M = ( 1,1) .
1 + xy
Prof. Amariei Laura-Georgeta, Colegiul Tehnic de Alimentaie i Turism
Dumitru Mooc Galai i prof. Amariei Lucian
19. S se determine a, b R astfel nct funcia f : R R dat prin f ( x ) = ax + b s fie
izomorfism ntre grupurile (R ,) i (R ,) , unde legea este definit prin

x y = x2 y 2 2 x2 2 y 2 + 6 .
Prof. Amariei Laura-Georgeta, Colegiul Tehnic de Alimentaie i Turism
Dumitru Mooc Galai i prof. Amariei Lucian
1
1
1
20. Demonstrai c
i apoi determinai a R tiind c
=

(x + 1)(x + 2 ) x + 1 x + 2
a

(x + 1)(x + 2 ) dx = ln 3 .

Prof. Amariei Laura-Georgeta, Colegiul Tehnic de Alimentaie i Turism


Dumitru Mooc Galai i prof. Amariei Lucian
Rubric realizat de prof. dr. Daniel Stretcu i
prof. Daniel Sitaru, Drobeta Turnu Severin

Probleme propuse_______________________________________________________________________________________ 40

INDICAII I RSPUNSURI
PROBLEMS IN ENGLISH

1.

Test tip BAC M2

Subiectul I
1.

,
,
, ,

2.
3.
4.
5.
6.
Subiectul II

1.a).
b) det(A+B)=0
,

c)

2.a) q=X-2,r=3X-6
b)
,
,
c)
Subiectul III
1.a) y=x asimptota oblica

b)
c) y-3x+4=0
2.a)
,
b)
CLASA A V-A

abc = 15321 + abc


100 abc = 15321 1021 <=> 100 abc = 14300

1. 100 abc + 21 + 1000 +

<=>
<=> abc = 143
2. a) Numrul de cifre este:9x1=9 cifre la numerele de o cifr
90x2=180 cifre la numerele de 2 cifre
900x3=2700 cifre la numerele de 3 cifre
2016-999=1017x4=4068 cifre la numerele de 4 cifre,
Deci n total vor fi 9+180+2700+4068=6957 cifre
b) 2016-(9+180)=1827 cifre pare din numerele de 3 cifre
1827:3=609 numerele de 3 cifre. Cum al 609-lea numr de 3 cifre este 99+609=708 => a 2016-a cifr este 8
3. Notm cu a,b,c i r numerele care trebuie determinate
=>a=55r,b=3c,c=3r =>b=9r i nlocuind n T.I.R. se obine 55r=27 r2+r=>27r2=54r i cum r 0 => r=2 =>a=110,b=18,c=6
4. Cum numerele 4x+1,4x+3 i 4x+5 sunt impare, iar B conine numere pare, sunt posibile cazurile
1) 5x-3=4x+2 => x=5
2) 5x-3=6x=>x=-3 nu aparine lui N
3) 5x-3=2016x-14080=>x=7,deci x{5,7}
Setul II
5. Fie numrul
divizibil cu 4 , 5 i 11 => b=0
Cum 3412 d restul 2 la mprirea cu 11 => a=2,deci numrul cutat este 341220
6. Deoarece [10,12,14,16,18,20] =5040 => numrul cutat este 2x5040=10080
9. Numerele sunt 12300 i 12375;
10. tim c (a;b) [a;b] = ab.
Obinem: a = 30 i b = 45 sau a = 15 i b = 90.
11. Fie (a;b) = d => ( ) x,y N, astfel nct a = dx, b = dy i (x;y) = 1.
Avem: [a;b] = dxy => d(1+xy) = 33.
Obinem perechile: (a;b) {(1;32), (6,15), (11,22)}

) (

5 2

) (

) (
2

= 185 185 = 185 121 + 64 = 185


11 + 8 = 185 11 + 185 8
13. Scznd cele 2 relaii => 2b-3a=68 i cu 2b i 68 sunt numere pare => 3a este numr par =>a este numr par. Dar a este numr prim=> a =2 =>b=37
i c=17
14. Fie x N*=>x=29xc1+r1,r1<29,c1=r1 =>
x=30c1 (1); x=35xc2 +r2,r2<35,c2=r2 =>
x=36c2 (2). Din (1) i (2) => 30c1=36c2 /: 6 =>5c1 =6c2
=> 6/c1 i 5/c2. Din 6/c1 i c1 < 29 => c1 {6,12,18,24}.
Pentru c1 =6 =>x=30 x6= 180;c1=12 =>x=30x12=360;
c1=18 =>x=30x18=540;c1=24=>x=30x24=720=>
x {180,360,540,720} (3)
Din 5/c2 i c2 <35 =>c2 {5,10,15,20,25,30}
Pentru c2 =5 =>x = 36x5=180;c2 =10=>x=36x10=360;
c2=15 =>x=36x15=540;c2=20=>x=36x20=720;
c2=25 =>x=36x25=900;c2=30 => x=36x30=1080 =>
x {180,360,540,720,900,1080} (4)
Din (3) i (4) =>x {180,360,540,720}
11

12. 185

10

10

41 ______________________________________________________________________________ Probleme propuse


15. Suma cifrelor numrului considerat este
1010 2 1010 4 2020 4040 6060
Rezult c numrul este divizibil cu 3.
Dac ar fi ptrat perfect ar fi divizibil i cu 9. Dar 6060 nu se divide cu 9.
Rezult c numrul nu poate fi ptrat perfect.
16. 100 abc + 21 + 1000 +
11

10

17. 185 = 185


18. X:25=a rest 10
X=25a+10
X1=250+10=10
X2=251+10=35

X40=2539+10=985
=>sunt 40 numere
19. I) A={a,b}
B={b,c}
20.
(6-4)
2
-2
=0

abc = 15321 + abc <=>

100 abc

= 15321 1021 <=>

100 abc

= 14300 <=> abc = 143 .

185 = 18510 (121 + 64) = (1855 ) (112 + 8 2 )== (185 5 11) + (1855 8)
2

II) A={a,b,c}
B={b,c}

III) A={a,c}
B={b,c}

IV) A={a}

B={b,c}

ab 2 = ...00 + ab
ab 2 - ab = ....00
ab ( ab - 1) = ....00
ab ( ab -1) = x 100
ab ( ab -1) = x 2 2 52
ab i ab -1 sunt numere consecutive

21.

=>Sunt prime ntre ele => unul din ele se divide cu 25.
Dac
Dac

ab
ab

= 25 => 625 = 252


= 26 => 676 = 262 nu verific

ab 2 = .....ab

Singura solutie este ab = 25.


,
, ..,
.
22. Numerele sunt
n=
.
n=

n=

n=

n=
=


+
=
23.

,
24. n este de forma 4k, 4k+1, 4k+2, 4k+3, K
.
Dac n=4k, U(
)=7
=7
Dac n=4k+1, U
Dac n=4k+2, U
=5 =>
se divide cu 5.
=1
Dac n=4k+3, U
Deci, 5|
dac i numai dac n este de forma 4k+2, k N.
25.

99
=

=
+
+
=
+

26.
Dintre numerele 1,2,3.....2009, fiecare al cincilea este divizibil cu 5, deci vor fi
divizibil cu

, deci vor fi

= 401 numere divizibile cu 5. Dintre aceste 401, fiecare al cincilea este

= 80 numere.

Dintre aceste 80 numere, fiecare al cincilea este divizibil cu

, deci vor fi

Dintre aceste 16 numere, fiecare al cincilea este divizibil cu 54, deci vor fi

= 16 numere.

2009
54 = 3 numere.

pentru c =3025.
Dintre aceste 3 numere, niciunul nu este divizibil cu
Atunci exponentul lui 5 din produsul 1x2x3......x2009 va fi 401+80+16+3 = 500.
27.
(a+1)(b+1) = 9, => a=2, b = 2
(1+p+p2)(1+q+q2) = 91
(1+p+p2)(1+q+q2) = 7x13 => p=2, q=3
N = 22x32 = 36.

Probleme propuse_______________________________________________________________________________________ 42
CLASA A VI-A
6. BC=49km, CD=18km; 7. 18 minute; 8. 24; 9. 34,5 cm; 10. 50 elevi.
11. Fie x si y msurile celor 2 unghiuri => x+y=180o i
=
=>x(180o-y)=y(90o-x) =>180ox-xy=90oy xy =>
Y=2x i dup nlocuire n sum => x=60o, y=120o
12. Din
= = = => a+b=6k,b+c=8k i c+a=4k.
nsumnd relaiile =>2(a+b+c)=18k=> a+b+c=9k =>a=k, b=5k, c=3k => 2k+15k+12k=87 =>k=3 =>a=3, b=15, c=9
13. Cazul I

Un triunghi ABC dreptunghic si isoscel, AB=AC. Inaltimea AD este si mediana si este din BC. Unghiurile vor fi
Cazul al II-lea

m(<A)=30 deoarece in triunghiul dreptunghic in D, BD= AB, deci unghiurile sunt

125
14. x-pretul initial;
x
100
15.

60 125
este pretul dupa majorare si
x
100 100

a) ABD si ACE- drept. isosc.


m(<BCA)=450(alt.int.) DE // BC

este pretul dupa ieftinire

ABCD paralelogram.
18.
=

60 125
x =2016
100 100

MCB.
900. Deci MNPQ este dreptunghi.
ca unghiuri alterne interne, rezult c DA BC. Deci

11 13
x ,
8 8

20.
=

x =(

)x

.
.
.
.
=
N= x(
N=

x =(
-

)x
+......+

x = 2688 lei.

m(<EAC)=450=

. Se obine k=3.

19.

D, A, E coliniare si

b) MCB-drept.isos. AC= inaltime si mediana [AB] [AM] si AE // BC AE=l.m in


16. Se demonstreaz c MNPQ este paralelogram i, din proporionalitate, rezult c m(
AB. Din

m(<DAB)=450 si m(<EAC)=450 m(<DAE)=1800

17. Utiliznd reciproca teoremei lui Thales se demonstreaz c DC

1 1
1

4 1 3 91 93

N= x
=
21. Din AB=z, BC=y i AC=y-z => ordinea C,A,B. Din AD=x, BD=x-z => ordinea A,B,D. Atunci avem ordinea C,A,B,D.
b) B este mijlocul segmentului AD =>AB=BD =>z=x-z =>x=2z. A este mijlocul segmentului CD => AC=AD => y-z=x=1 => y-z=2z => y=3z.
=>

43 ______________________________________________________________________________ Probleme propuse


26. a=65 +39; a 13; numerele cutate sunt 234 i 299; 27. 6x+11y+3z=50, 50 5; 28. a=

29. n propoziia dat am putea egala numrtorii i numitorii ntre ei,numai dac fracia

ar fi ireductibil.Fie d=(a2-a,2a-1) => d/a2-a i d/2a-1 =>d/2(a2-

a) i d/a(2a-1) => d/2a2-a-2a2+2a=> d/a => d/2a i d/2a-1 => d/2a-2a +1 => d/1 => d=1 ,Deci fracia
<=> a(a-1) =9x8 => a=9
30. a {2,3,5,7,11,17}
(a,b)
,
,
,
31. a=2, c=3, b=5
=3
S=2+
32. Numaratorul si numitorul trebuie sa se divida cu 4 si 11 care sunt numere prime intre ele.
b
,
d
,
daca [(a+2)-(1+1)] 11a=0
b=2
daca [(2+3+d)-(c+5)]
, daca d=2 [(5+2)-(c+5)]
c=2
=22352
pentru a=7
b=6

=26356
d=6
,
,
,

sunt
33.

3212 - 9 6 = (29 + 3) 12 -3 12

=M 29 + 3

12

-3

12

;
este ireductibil => 2a -1=17 => a=9 i a2-a =72

= M 29 M 29.

34. Fie x msura celui mai mic unghi.


Atunci: x + (x+60) + (x+ 260)+ +(x+ (n-1)60) = 360
<=> nx + 60

(n 1) n = 360 <=>
2

nx + 30(n-1)n = 360 <=>

<=> n[x+30(n-1)] = 360.


Pentru n = 2 => x = 150 i x + 60 = 210.
Pentru n = 3 => x = 60, x + 60 = 120 i x + 120 = 180.
35. 16 16
36. Fie

16 3 ... 16 2015 = 161+ 2+...+ 2015 = 1610082015 1610082015 =(16504x)2 x=2015

cum | rezult |
.
37. Din a + b + c = 1998 i b c = 42 (adunnd cele doua relaii), rezult c
a +2b = 2040
Dar 2b
i 2040 , rezult a i a este numr prim => 2 + 2b = 2040 / -2 => 2b = 2038 => b=1019
Din b-c=par =>b+c =par i din a + b +c = 1998 rezult a = par => a = 2, atunci c = 997.
38. Fie n numarul cutat
Din teorema mpririi cu rest obinem obinem:
n = 28c1+7 => n 7 = 28c1
n = 35c2 +7 => n 7 = 35c2
n = 40c3 +7 => n 7 = 40c3
n = 70c4 +7 => n 7 = 70c4
Rezult c n 7 este multiplu comun al nemerelor 28, 35, 40, 70.
[28,35,40,70] = 280
Deci n 7 este M280 cuprins ntre numerele 250 i 1200 => n
,
,
,
=> n
,

39. a)

x= p.p

1 1 1 1 1
1
1
3
( + + ... +
)<

2009 3 7 7 11
2015 2019
2019
1 1
1
3
1
672
3
672 3
(
)<

<

< (A)
2009 3 2019
2019
2009 2019 2019
2009 1
2 1008
41. (7 )
+ 7 2016 = 7 2016 + 7 2016 = 7 2016 2 (1)
2 1008
(5 ) + 2 5 2016 = 5 2016 + 2 5 2016 = 5 2016 3 (2)

40.

Din (1) si (2)


42.

1
1
+
2 2 32
1
1
+
2 2 32

cmmdc=1 491008 + 7 2016 si 251008 + 2 5 2016 sunt prime intre ele.


1
1
1
1
1
1
,
,,
si
<
<
<
22 1 2
32 2 3 2016 2 2015 2016
1
1
1
1
1
2015
+ ... +
<
+
+ ... +
= 1
=

2015 2016
2016 2016
2016 2 1 2 2 3
1
2015
+ ... +
<
2016 2 2016

43.
111=110+1
n=1900+110(110+1)-1102(110+1)+1103(110+1)-1104(110+1)++
11011(110+1) -11012(110+1)+11013=1900+110=2010=6730.

adunand

se

obtine

Probleme propuse_______________________________________________________________________________________ 44

x +1 x + 2
x + 2015
x +1
x+2
x + 2015
+
+ ... +
= 11
+4
14
+2
1 +4
...4
+
31 ( 2 1) + ( 3 1) + ... + ( 2016 1) = 0
2
3
2016
de 2015 ori
1 1
1
( x 1)( + + ... +
)=0
2 3
2016

44.

x-1= 0 x=1

n + 2012
1
= 1
n + 2013
n + 2013
n + 2013
1
= 1
n + 2014
n + 2014
1
n + 2015
= 1+
n + 2014
n + 2014
1
n + 2016
= 1+
n + 2015
n + 2015

n + 2012 n + 2013 n + 2016 n + 2015


<
<
<
n + 2013 n + 2014 n + 2015 n + 2014

45.

46. tim c (a;b) [a;b] = ab.Obinem: a = 30 i b = 45 sau a = 15 i b = 90.


47. Fie (a;b) = d => ( ) x,y N, astfel nct a = dx, b = dy i (x;y) = 1.Avem: [a;b] = dxy => d(1+xy) = 33.Obinem perechile: (a;b) {(1;32), (6,15),
(11,22)}.
48.

3212 - 9 6 = (29 + 3) 12 -3 12

=M 29 + 3

12

-3

12

=M 29 M 29.

49. Fie x msura celui mai mic unghi.


Atunci: x + (x+60) + (x+ 260)+ +(x+ (n-1)60) = 360<=><=>n[x+30(n-1)] = 360.Pentru n = 2 => x = 150 i x + 60 = 210.Pentru n = 3 => x =
60, x + 60 = 120 i x + 120 = 180.
50. 19|2a+57 i 19|57 => 19|2a => 19|a => 19|5a.
22=19+3 => 5a+22=5a+19+3
19|5a+19 => 5a+19=19k, 5a+22=19k+3 => restul mpririi la 19 este 3.
51. xy=[x,y](x,y)
xy=15(x,y)2 => (x,y)2=135:15 => (x,y)2=9 => (x,y)=3.
[x,y]=15x3 => [x,y]=45.
(x,y)=3 => x=3a, y=3b, (a,b)=1.
xy=135 => 3ax3b=135 => ab=45.
a) a=3, b=5 => x=9, y=15
b) a=1, b=15 => x=3, y=45.

n (n + 1)
n (n + 1)
n (n + 1)
=>
= xxx =>
2
2
2
2 3 37 x => n(n+1) = 6 x 37 6 x = 36 n = 36 .

52. 1+2+3++n =

111 x

=> n(n+1)=

= 222 x

=> n(n+1) =

53.
Notm m(<AOC)=x => x+(x+1)+(x+2)+(x+3)+(x+4)=180 =>
5x+10=180 => 5x = 170 =>x=34 => m(<AOC)=34, m(<COD)=35, m(<DOE)=36, m(<EOF)=37, m(<FOB)=38.
CLASA A VII-A
1. Din 2a+3b=5c =>2a-2c=3c-3b,adic 2(a-c)=3(c-b)=>2/(c-b) i 3/(a-c) (1).Din 2a+3b=5c => 3b-3a=5c-5a,adic 3( b-a)=5(c-a)=>3/(c-a) i 5/(b-a) (2).
Cum(2,3,5)=1,din (1) i (2) =>30/(b-a)(c-b)(a-c).
2. (a- +(b-2 +(c-3 = -2a +3+ -4b +12+ -6c +27= + + 2 (a+2b+3c)+42= + + -2 7 +42= + + .
3. x 1x=1x
4. Se obine n=9 care este ptrat perfect.
5. Mediana mparte un triunghi n dou triunghiuri echivalente.
A ABM = 104 cm : 2 = 52 cm . A ABN = 52 cm : 2 = 26 cm .
N - mijlocul lui [AM], iar NP || BC cu P (AB) => P mijlocul lui [AB]
2

=> A BPN = 26 cm : 2 = 13 cm .
2

Sau: APN ~ ABM => A APN = A ABM : 4 = 52 cm : 4 = 13cm


2

=> A BPN = A APN = 13 cm .


2

6. Daca ABCD paralelogram si AM= AD, CN= BC atunci MBND paralelogram.


In ABM , AM = AB, m( A) =60o se obine m( M) =90o deci m( < BMD)=90o.
=> MBND dreptunghi..
7. a) Din punctele D si E simetricele punctelor B, C fata de A=> Ea=AC si DA=AB => EDCB paralelogram.
b) Pentru ca EBCD sa fie ptrat trebuie ca diagonalele sa fie perpendiculare si congruente, deci ABC trebuia sa fie dreptunghic isoscel.
8.
nsumnd toate relaiile pentru n=1,2,3,..,79 se obine

<2

45 ______________________________________________________________________________ Probleme propuse


9.

=2.

10. Din 2a+3b=5c =>2a-2c=3c-3b,adic 2(a-c)=3(c-b)=>2/(c-b) i 3/(a-c) (1).Din 2a+3b=5c => 3b-3a=5c-5a,adic 3( b-a)=5(c-a)=>3/(c-a) i 5/(b-a) (2).
Cum(2,3,5)=1,din (1) i (2) =>30/(b-a)(c-b)(a-c).
11. ( -2x +7)+( -2y +6)=0 <=> (x- +(y- =0 =>x= i y= =>( + )( - )= ( + ) ( - )=7-6=1

12. Se aplic inegalitatea mediilor


3

,oricare ar fi a,b

Prin nsumare avem:3


+4
+ 5

Egalitatea are loc dac: 9=x-9 => x=18 , 16=y-16 =>y=32 , 25=z-25 =>z=50
13. Se scrie inegalitatea:
2

i se adun inegalitile. Obinem:

, , ,,

pentru

18

Dar

14.

. Rezult

de unde

18
18

10101012 (13 2 12 2 )
10101012 (7 2 6 2 2 2 )

10101012 25 1010101 5 5
=
= Q
10101012 9 1010101 3 3

15.

m (< B ) = 1800 600= 1200; m (< A ) = 180 m (<B) = 600


BC =

2
AB = 8
3

A = AD AB sin 600= 8 12
AM = 6 cm si in

3
= 48 3
2

ADP drept. cos600=

In DPM drept.
16. Se observ c :

dar A = ABDP

DP = 4 3

AP
AP = 4cm si PM =2cm
AD

DM = 2 13 cm si P = 24+ 16 = 40

17. Din (1) i (2)


Notm cu , ,
Deci vom avea:

lungimile laturilor triunghiului, iar

aria triunghiului.

Dar
mprind prin

, deci:
n inegalitatea de mai sus vom avea:

2 Fals

Probleme propuse_______________________________________________________________________________________ 46
n concluzie, rezult c nu exist triunghiul respectiv.
|
|
18. Cum
I.
Dac
II.
Dac
19.

Deci, soluiile sunt

Observm c :

Cum punctele

, ,

sunt coliniare n

i transversala

este mijlocul lui

avem:

este median n

20. Putem scrie :

Si tinand cont ca in
Iar

avem

Si tinand cont ca in
Deci o sa avem:

avem

21. Dupa ce se inmultesc parantezele vom avea :

Putem scrie

Si vom avea :

Deci putem scrie :


sau
Dar cum cele trei numere sunt nenule atunci prima ecuatie este imposibila.
Solutiile sunt:
, ,
, , ;
, , ; ,
, ; , ,
;
22. Dupa inmultirea parantezelor vom avea:

, ,

Vom avea:
I.

nu avem solutii in X

sau

II.

,
sau
nu avem solutii in X

Din I, II avem :
,
23. Se nmulete relaia dat cu 2 i se aranjaz sub forma

a = 2, b = 1 i expresia cerut are valoarea -1.


24.
28.

(a 2)2 + (a 2b)2

= 0 . De aici se determin

21 8 5 = 4 5 ,17 + 12 2 = 3 + 2 2 ,12 8 2 = 2 2 2 .

47 ______________________________________________________________________________ Probleme propuse


Mediana mparte un triunghi n dou triunghiuri echivalente.A ABM = 104cm : 2 = 52cm . A ABN = 52 cm : 2 = 26 cm .N - mijlocullui
2

[AM], iar NP || BC cu P (AB) => P mijlocullui [AB] =>A BPN = 26cm :


2

2 = 13cm .Sau: APN ~ ABM =>A APN = A ABM : 4 = 52cm : 4 = 13cm


2

=>A BPN =A APN = 13cm .


2

29. Daca ABCD paralelogram si AM= AD, CN= BC atunci MBND paralelogram.In ABM , AM = AB, m( A) =60o se obine m( M) =90o deci m( <
BMD)=90o.=> MBND dreptunghi.
30. a) Din punctele D si E simetricele punctelor B, C fata de A=> Ea=AC si DA=AB => EDCB paralelogram.b) Pentru ca EBCD sa fie ptrat trebuie ca
diagonalele sa fie perpendiculare si congruente, deci ABC trebuia sa fie dreptunghic isoscel.
31.

=2
CLASA A VIII-A

1. U( )=1, U(
)=3, U(
)=9, U(
)=7 =>
U[(
)+. . . . +(
+
+
+
)+(
Deci numrul de sub radical nu este ptrat perfect.

2. a)(

b) )(

c) (
3. E(x)=

)]=U(0+0+. . . .+0+3+9)=2 ,

4. Pentru m=5 ecuatia este nedeterminata deoarece 0x=x x


Pentru m=4 ecuatia este imposibila deoarece 0x=1
Pentru m4 si m5 ecuatia este compatibil determinata cu solutia x=
5. Daca x
\ ,
x= .
6. V(ABCD) =V(ABCF) +V(ABFD). Aria bazei comuna a piramidelor in care am descompus piramida data ABCD este A(ABF) =100

=1
CD(ABF) V(ABCD) =

=1 l

7. Fie a,b,c dimensiunile


a=2k, b=3k, c=5k
=2(2 3k+2k

=248

8.

=4k=2a=4cm, b=6cm, c=10cm

Analog:

1
1

3
;1
4
1

1
1

3
4
27
64

8
64

1
8

1
1
2
x4 2 + 4 = y2 x4 + 4 = y2 + 2
2 1
2
x
x
9. Notm
=> x 2 = y =>
=>
x

2
2
y + 2 + y = 2 <=> y = 0 <=> x 2 1 = 0 <=> x 2 = 1
Ecuaia devine:
x2
x2
4
<=> x = 1 <=> x = 1 .

x2

1
=y
x2

10. Din x+y+z=12 i xy+xz+yz=48 =>( x+y+ -3(xy+xz+yz)=


-3x48=0 =>
-xy-xz-yz=0 /x2 =>
-2xz-2xy-2yz=0 <=> (x0
0
0
11. a)Se impun condiiile:7-3x0 i 6+3x0 =>x [-2, ] (1)

+(x- +(y

12

Suplimentar : 7-3x 25 i 6+3x 25 => x [-6, ] (2)


Din (1) i (2) =>x [-2, ]. Prin ridicare la ptrat =>
7-3x +6+3x+2
1)(3x+2)=0 =>x1=1 i x2=

=25=>

=6 <=> (7-3x)(6+3x)=36 <=> 3x2 x-2=0 <=>3x2-3x+2x-2=0 <=>3x(x-1)+2(x-1)=0 <=>(x-

,ambele rdcini aflndu-se n intervalul [-2, ].

b)Din condiiile 4x+1 0,6-x0 i 2x-30 =>x [ ,6] (1)


Prin ridicare la ptrat se obine:
=2x-3 =>2
=x+10
4x+1+6-x-2
Suplimentar:
Se
pune
condiia
x+100
=>
x-10
(2).
Din (1) i (2) => x[ ,6]. Se mai ridic la ptrat i se obine 17x2-72x+76=0 <=> 17x2-34x-38x-76=0 <=> 17x(x-2) -38(x-2)=0 <=>(x-2)(17x-38)=0
=>x1=2 i x2= ,ambele rdcini aflnduse n intervalul [ ,6].
12. inegalitate mediilor:

,x,y R+ 3 =1 3

Probleme propuse_______________________________________________________________________________________ 48
4

2014= 1

2015= 1

3 +4
13.

2014+5

2014

2015

.nsumm

inegalitile

2015

obinem

=6x+2016

=>

14.

Rezult:

15. Fie P(x) polinomul care este membrul stang al inegalitatii din problema.
,
,
;
P(x)>0
Daca x (-,0] atunci >0,
0si 1-x>0
Daca x (0,1) atunci P(x)=
0
0 deoarece
0,
si
Daca x [1,) P(x)=
16. E(a)=a(
[(
. Produsul a patru numere consecutive se divide cu 4. Deci E(a) se divide cu 4.
17.
|
| |
|
Pentru x
, ecuatia devine 5-x+x-3=2 adevarat
Pentru x
,
5-x+3-x=22x=6, x=3 nu convine deoarece x>3
Pentru x=5 2=2 adevarat
Pentru x
, x-5+3-x=2 fals x
,
18. Ecuatia se poate scrie (x+y)(x-y)+(x+y)=2(x+y)(x-y+1)=2
Avem

0,

=(

cu solutia (x,y) {(1,0)}


cu solutia (x,y) {(-2,1)}
cu solutia (x,y) {(1,1)}
cu solutia (x,y) {(-2,0)}

(x,y) {(1,0), (-2,1),(1,1),(-2,0)}


19. F(x)=
20. E(x)=

=(x-1)(x-2) x=3 si F(3)=2

=
=5+

=5+

Emax= E(2)=5+1=6

1
1
1
1
1
1
1
1
1
1
1
1
3

+ +

21. a=
=
=
504 508 508 512 512 516
2008 2012 2012 2016 504 2016 2016
2
4
1
1
;
(
;
) a (
).
2016 2016
1008 504
1
1
1
2
x2 2 = y
x4 2 + 4 = y2 x4 + 4 = y2 + 2
1
2
2
x
x
x
22. Notm
=> x 2 = y =>
=>
x

2
2
y + 2 + y = 2 <=> y = 0 <=> x 2 1 = 0 <=> x 2 = 1
Ecuaia devine:
x2
x2
4
<=> x = 1 <=> x = 1 .

49 ______________________________________________________________________________ Probleme propuse


CLASA A IX-A
;

1. Fie

;
;

2.

3. a)6 ,b)1 ,c)1,d)0.


4. Din inegalitatea mediilor
5.

Din

condiiile

1 2x

de

1 + 2x
2

existen

etc.

radicalilor

rezult

x y z + 2 = 0.

Egalitatea

din

enun

devine:

3 y + x + 2 8 x + 6 y = 14

(3

6.

x + y + 2 8 x + 6 y (32 + 2 2 + 12 )( x + y + 8 x + 6 y ) = 14 2 .
2

n+1

1n k
(10 1) + n 1 = 10 + 72n 91.
Sn = 1 + (1 + 11) + (111+ 1) + ... + 111
12...
31 + 1 Sn = 9
81
k =1

nori

7. .

. Insumnd toate relaiile pentru n=1,2,3,..,79 se obine

<2.
,
,
.
8.
9. Prin restrngere, ecuaia se scrie
10. Se utilizeaza relaia lui Hermite i se obine soluia
12. a) Fie

raportul n care punctul

mparte segmentul

.Aplicnd teorema lui Menelaus n


(sunt vectori de acelai sens)

, de unde se obine soluia

cu transversala

.Se aplic teorema lui Ceva i se obine


, se obine
Din relaiile

.b)

.Din relaiile

.
13. sir monoton si marginit.
14. inegalitatea mediilor si formula termen general/proprietati progresii.

(n + 1)2 , ()n N*
<=> n <
n (n + 1) < (n + 1) => [ n (n + 1) ] = n
n(n + 1)
Ecuaia devine: 1 + 2 + + n = 45 <=>
= 45 => n = 9.

15.

n2

<

n (n + 1)

<

18.

+ n = 45

n (n + 1) < (n + 1)
n(n + 1)
<=>
= 45 =>n = 9.
2
<

()n N*. <=> n <

n (n + 1) < (n + 1) =>[ n (n + 1) ] = n . Ecuaia devine:

1+2+

Probleme propuse_______________________________________________________________________________________ 50
19. Notm

x2

1
1
1
= y . Ecuaia devine: y 2 + 2 + y 2 = 2 <=> y = 0 <=> x 2 2 = 0 <=> x 2 = 2
x2
x
x

<=> x

= 1 <=>

x = 1 .
| | , se obine c numrul dat este egal cu 3, deci e natural.
20. Prin trecerea la modul (
21. Se utilizeaz formule de calcul prescurtat (binom sum/diferen).
,
,

22. Explicitnd modulele, se obine

,
,

Intersecia cu axa Ox: punctul de intersecie dintre grafic i axa Ox are coordonatele
grafic i axa Oy are coordonatele ,
. b) Prin calcul se obine
3 i

coordonate ,
se gsete pe graficul funciei.
23. a) din conditiile compunerii functiilor; b) S=

.Intersecia cu axa Oy: punctul de intersecie dintre


, rezult c punctul de
. Cum

n
.
3(2n + 3)
CLASA A X-A

1. |

| |

| |

| |

| |

| |
| |

| |
| |

| |
| |
2.
i fie
, ;
, ;
|
|
|
;|
;
i ecuaia se scrie
.
Rezult , , coliniare;
de unde
,
3. Fie
;
, . Rezult:

,
;
;

| |

| |

| |

| |

If

,,

, then:

4. Observm c

f (1) = f (2014) = 0

5. Se ridic la puterea a 3-a si obtinem:

i deci funcia f nu este o funcie injectiv.

)(

x + x 2 + 27 + x x 2 + 27 + 23 x + x 2 + 27 x x 2 + 27 = 4

2 x + 2 27 = 4 de unde solutia.
6. Se arat uor c funcia f este injectiv i surjectiv.
7. Se amplific partea stang cu x+2-(y+4)i si se aduce la forma a+bi ,de unde a=0 si b=1.
3

2015

k =2

9.

1
1
1
2015
k ( k +1)

k +1
k ( k +1)
k
(
2015
)
=
2015
= 2015 k = 2
= 2015 2 2016 > 2016 2015

2015

8.

| | = | z 2015 |=| z | 2015 i | z |=

1
= 1 | |= 1 .
1

i deci fracia este supraunitar.

k =2

cos 2015 + i sin 2015


cos 2015 + i sin 2015
=
=
sin 2016 + i cos 2016
sin 2016 + i cos 2016

51 ______________________________________________________________________________ Probleme propuse


10. Avem
=

(a

11.

= lg 2015 lg 20150 lg 201500 lg 2015000 + 1 = a(a + 1)(a + 2)(a + 3) + 1 =

+ 3a + 2

) , unde a = lg 2015 .
2

f ( x) = a x a +

12.

1 10
=
a 3

Vom pune conditiile de existen a radicalilor :

x2 5

,obtinem

a > 1 , deci a = 3..

5 5 x 0

x0

5 x 0

apoi se ridic la ptrat i obinem

5 x = 5 x2

5 5 x 0

x0

x0
5

5 x2 0

n continuare vom ridica din nou la ptrat i obinem ecuaia


considera drept variabil numarul 5si vom obine :

x 4 2 5 x 2 + 52 + x 5 = 0

vom rezolva o ecuaie de gradul II n care vom

5 5(2 x + 1) + x + x = 0 5 = 4 x + 4 x + 1 4 x 4 x = 4 x 4 x + 1 = (2 x 1)
2

vom pune condiia

Notam 5=a

2x 2 + 1 2x + 1
x2 x +1 a = 0 x2 x 4 = 0
2
2x 2 + 1 + 2x 1
a2 =
x 2 + x a = 0 (2) x 2 x 4 = 0
2
a1 =

(1)

Am obinut dou ecuaii de gradul al II-lea in variabil x


La ecuaia (1) calculez

x1 =

1 = 4a 3 1 = 17

de unde

x1 =

1
3
1
3
+ a , x2 = a
2
4
2
4

3 1 + 17
3 1 17
1
1
, x2 =
+ 5 =
5 =
4
2
4
2
2
2

La ecuaia (2) calculez

1
1
2 = 1 + 4a 2 = 21 de unde x3 = + a +
2
4

1
1
x4 = a +
2
4

1
1 1 + 21
1
1 1 21
x3 = + 5 + =
=
, x4 = 5 +
2
4
2
2
4
2
2015

13.

2015
a 2 + bk2
a k + ibk
= k
= 1.
k =1
bk2 + a k2
k ia k

b
k =1

14.

(1 + i ) 2014 z + 2 2014 1 + i

15.

z=

2014

z + 2 2014 ( 2 ) 2014 + 2 2014 < 2 2014 + 2 2014 = 2 2015

m +1
(m + 1) (m 1 mi) m 2 1 m(m + 1)i
R z =
=
R m = 0, m = 1
m 1 + mi
(m 1) 2 + m 2
(m 1) 2 + m 2

16. Avem dou situatii


a)
b)

m 1
m 1
m 1
< 1 si avem x 2 + 2010
x 2 + 2010
0
m +1
m +1
m +1
1

m 1
m
m
1
> 1 avem x 2 + 2010
x 2 + 2010
0 .
m +1
m +1
m +1
0<

17. Se arat uor c funcia f este injectiv i surjectiv.


18.

f ( ) =

= =

1
f ( )

Probleme propuse_______________________________________________________________________________________ 52
x +1
2

1
1
3 x 1 = 3 x x =
2
3x
2
20. lg( x + 1) + lg( x 1) = 2 lg 3 + lg 11 lg( x 1) = lg 99 x = 10

19.

21. se amplific partea stang cu x+2-(y+4)i si se aduce la forma a+bi de unde a=0 si b=1
22.

z1 = x + iy , z 2 = u + vi z1 z 2 z1 z 2 = xu + vy + i(xv - yu) - xu - vy - i(yu - xv) = 2i(xv - yu)

23.

rezulta ca
0

24.

x y + 2 xyi = 3 + 4i

(x+iy) =3+4i
2

x2 y2 = 3
4

2 x 2 2 = 3 x 4 3 x 2 4 = 0 clz
2
=
4

=
2

=
xy
xy
y
x

x
25.
3

a a b b a + ab + b
a 2 b 2 a + a 2b 2 + b
:
+ 2a 2 b 2 =
+2 a b =
E= 1
:
2
2
1
a
b

a+ b
a
b

2
2
a +b
1

a + a 2b 2 + b
a a b b a + ab + b
a b
+ 2a 2 b 2 =
:
+2 a b =

2
2
ab
a+ b
a+ b
a b

)(

a b

a+ b

a + ab + b

+ 2 a b = a + b
26.
a)
b)
c)
d)
e)

Condiii
, . Se obine
,
.
Condiii
,
.Se obine
,
.
Condiie de existen
. Rezult
, .
Se analizeaz cazurile
i
0 . Se obine
Condiie de existen
. Se obine
,

1. Determinantul numitorului este


Dac

, .

CLASA A XI-A

= a ( a3 + 8) .

< 0 2 x 2 + a 2 x a 0, x

lim f ( x)
x b
x <b

finit (egal cu

f (b) ) , pentru orice b

2
2
= 0 a = 0 sau a = 2 care conduc la f ( x) = 4 x 1 sau f ( x) = 4 x 1
2
2
2x
2 ( x + 1)
b = 1 .
1
1
Dac > 0 se impune ca ecuaiile s aib o unic rdcin real comun (
sau ) , ceea ce implic
2
2
a = 1 sau a = 1 2 .

Dac

Pentru

a = 1 f ( x) =

( 2 x 1)( 2 x + 1) = 2 x + 1 , cu b = 1
( 2 x 1)( x + 1) x + 1
.

Pentru

a = 1 + 2 f ( x ) =

Deci

, cu b = 2 1

, cu b = 1 + 2

2 x + 2 +1

Pentru

a = 1 2 f ( x ) =

2x 1

2x 1

2 x +1 2

a 0, 2,1, 1 2, 1 + 2

cu

, nu convine !

b=0

respectiv

53 ______________________________________________________________________________ Probleme propuse

1
1

2 AB 4 A B = O2 (1) A I 2 ( B 2 I n ) = I n ( B 2 I n ) A I n = I n
2
2

2 BA 4 A B = O2 (2) .Din (1) i (2) AB = BA. Cu o proprietate cunoscut det ( A2 + B 2 ) 0 .

2. Din enun

1
1
a
b < 0 , ecuaia bx 2 + 1 = 0 are rdcinile reale opuse x1 = , x2 = + .Observm c
+ c < 0 .Avem
b
b
b
a

f ( x1 ) = x1
+ c > 0, iar f ( x2 ) = x2
+ c < 0 , unde f ( x) = ax 3 + bx 2 + cx + 1, f :
b

continu.Ecuaia f ( x ) = 0 are astfel trei rdcini reale situate respectiv n intervalele ( , x1 ) , ( x1 , x2 ) , ( x2 , ) .


3. Pentru

4. Notnd cu
Dac

limita dat , avem:

0 < a < 18 L = .

1
2
3
L = lim x 2 + + 8 + a + =
x
x
x
x

18 a

a > 18 L = Dac a = 18 L = lim 2 x + 1 + 2 2 x + 1 3 2 x + 1


x
2
4
6

1
1
1
1
L = lim 2 x + x + + 2 2 x + x +
x
2
6
4
6

1 1
1 1

2 6
4 6
L = lim 2
+ lim 2 2
= 0+0 = 0.
x
1
1 x
1
1
x+ + x+
x+ + x+
2
6
4
6

Dac

5.

a)

Prin calcul direct.

b)

Folosim a) , i avem egalitatea

( x + y)
ntruct

det ( A + I 2 ) + det ( A I 2 ) = 2 (1 + det A ) .ns utiliznd inegalitatea

4 xy x, y

, rezult 4

(1 + det A)

4 det ( A + I 2 ) det ( A I 2 ) .

det ( A + I 2 ) det ( A I 2 ) = det ( A + I 2 )( A I 2 ) = det ( A2 I 2 ) , se va ajunge la inegalitatea cerut.

6. Demonstrm prin inducie dup

c
;
!

!
!
!

!
!
!

!
!

ir constant deci convergent.


;| |
, ,

!
!

!
!

!
!

| |

| |
Fals!

8.

. Se presupune adevrat
. De demonstrat.

7.

!
!

Probleme propuse_______________________________________________________________________________________ 54
!
!

21.

Se demonstreaz c

B n = O3 , ()n 3 . Se calculeaz An

folosind formula binomului lui Newton.

cos 3 x sin 3 x
i se aplic formula cos 3 x = 4 cos3 x 3 cos x .
22. Se arat c A =
sin 3 x cos 3 x
3n 1
xn =
2n + 1 .
23.
24. Se demonstreaz prin inducie matematic.
25. Se utilizeaz conjugatul termenului general i se demonstreaz c 0 < xn 3 .
14. Deoarece f continu n x0 = 1 lim f ( x ) = lim f ( x ) = f ( 1) .
3

x 1
x <1

Avem

,iar

x 1
x >1

1
1
3
3

lim f (x) = lim b2 x3 + ax + = a b2 + , lim f ( x) = lim a2 x2 + bx + = a2 b +


x1
x1
x1
4
4
4
4

x<1
x >1
x >1

x1
x<1

f (1) = a + b + c .Rezult c a b 2 +
2

1
3
1
= a 2 b + a 2 + a + b2 b =
4
4
2

1
1
3
1
1
1

a + + b = 0 a = , b = .Din a 2 b + = a + b + c c = .
2
2
4
2
2
2

sin x tg 2 x 3x
3x 2
2 2
sin t
tgt
t
15. Avem lim

= 11 1 1 = , cci lim
= lim
= lim t
= 1.
x 0
x 0
x0 t
x0 e 1
x
2 x e3 x 1 e3 x 1 9
9 9
t
2
16. Separat cos 4 x 1 = 2sin 2 x (formul trigonometric)
ln ( 3 x 2 + 1)
ln ( 3 x 2 + 1) 2 x
2 x 1 3 x 2
1 3 3
lim
= lim


= = 1 1 1 =
2
x 0 2sin 2 x
x0
3x 2
sin 2 x sin 2 x 2 4 x 2
2 4 8
ln (1 + t )
t
, cci lim
= lim
= 1.
t 0
t 0 sin t
t
3 2 3 2 p 2 p q 0
1
17. a) A =

=
+
p = 1, q = 4 .
2 1 2 1 2 p 3 p 0 q
Alternativ, se verific faptul c
b)Nu exist ! Dac ar exista

X M2 (

A2 4 A = I 2

i nmulind cu

A1 A1 = A 4 I 2 p = 1, q = 4 .

det ( X X T ) = det ( A ) det ( X ) det ( X T ) = 1 ( det X ) = 1 , fals pentru


2

).

18. a)Prin calcul direct (Sarrus)

D ( a ) = 2a 3 + a 2 + 8a 12

(1). Apoi

( a 2 ) ( a + 3) = ( a + 4a 4 ) ( a + 3) = 2a + a + 8a 12 (2).Din (1) i (2) , va rezulta cerina.


2

b)folosind a) ,ecuaia devine

( m 2 ) ( m + 3) = 4 ( m 2 ) ( m + 3) = 4 , cu m 2, m + 3 numere ntregi.
2

Divizorii de tip ptrat perfect ai lui 4 sunt 1 i 4.

( m 2 ) = 4, m + 3 = 1 ,nu convine!
2
Cazul ( m 2 ) = 1, m + 3 = 4 , conduce la
Cazul

c)Folosind b)

m = 1.

( 2 x 8 ) ( 2 x 3) = 0 2 x = 8 sau 2 x = 3 x {3, log 2 3} .

19. Relaia este echivalent cu

2 x 2 + 2 x 4 = 0 , care admite soluiile x1 = 1 i x2 = 2 .

20. Se calculeaz A B i se arat c det A B = 0 .


21. Se arat c domeniile de definiie pentru cele dou funcii nu coincid.

55 ______________________________________________________________________________ Probleme propuse

sin x
= 1.
x
23. Din egalitatea ls (1) = ld (1) se obine a = 4 .

22. Se aplic formula

lim

x 0

CLASA A XII-A
este un izomorfism de grupuri.

1. Funcia : ,
, ;
Dac
atunci exist
;

Din
rezult
n aceste condiii:

2.

If

3.

Rezult:

3
4.

3
2

2 33

, on

, then
;

Probleme propuse_______________________________________________________________________________________ 56

5.

6.
a)

| |

b)
c)

+C

d)

7. a)1,b)x=2,c) x+y=y+x deci

9. a) calculeaza
=1 .

.
10. a) Se verifica axiomele; b) x*2011=2011 pentru orice x numar real si se foloseste asociativitatea legii cf.a). 11. Din condiia de continuitatepe R se obtine
a=163/12 si b=-55/4.
12. a) calcul; b) legea nu esteasociativa; c) x=2015; d) se alege un nr a din R\ Z si aflam b astfel nct compunerea lui a cu b este numar intreg (fixat).
13. Se verifica condiiile: f derivabil i derivata funciei f este funcia g. b)O functie este crescatoare daca are derivata pozitiva, problema revine la a arata
e

ca f>0pe [1, ]; d)conform a) integrala devine

f ( x) f '( x)dx =
1

f 2 ( x)
.
2 1

14. a) se verifica axiomele grupului. b) a=b=1.


3

e - 1 e - 2
; b)
3
2e 2
16. Se arat c x y = ( x + 1)( y + 1) 1
15. a)

ln y

ln x ln y

i se obine rezultatul

1.

x
=e
.
x y 1 < 0 i x y + 1 > 0 .
19. Din relaia de morfism se obine (a, b ) {(0,2 ); (0,3 ); (1,2 )} , iar din condiia de bijectivitate se obine a = 1 i b = 2 .
17. Se utilizeaz relaia

18. Se arat c

20. Se obine prin calcul.

57 ______________________________________________________________________________ Probleme propuse

RUBRICA REZOLVITORILOR

Listele cu elevii evideniai n rezolvarea problemelor vor fi trimise de cadrele


didactice oricruia din membrii colectivului redacional.
Materialele propuse spre publicare vor fi de asemenea transmise oricruia din
membrii colectivului redacional.
Elevii evidentiati in rezolvarea problemelor din revista Minimath nr. 2/2014:
C.T.DL.TUDOR , DR. TR. SEVERIN
Clasa a XI-a A: Mitvan Ionut, Girbovan Vladut, Panait Paul, Modilca Daniel, Truican Ionut, Cirtog Mihai (Prof. Magda
Patuleanu)
Clasa a X-a C: Nanora Robert, Bogasieru Costinel, Burca Cristian, Enea Alin, Odor Andreea, Vladimirescu Mihai (Prof.
Magda Patuleanu)
Clasa a IX-a A: Dilganu Laurentiu, Dilganu Daniel, Brindusescu Claudia, Gheorghisan Costinela, Botgros Puiu (Prof.
Magda Patuleanu)
COLEGIUL NAIONAL GHEORGHE IEICA
Clasa a VIII-a: Brbu Denis, Bora Ana Maria, Brehui Ana Maria, Cheredi Doru, Ciochia Andrei Toni, Coard Daria
Maria, Combei David Gabriel, Costache Patricia tefania, Deac Drago tefan, Drghici Daniela, Fril Coravu
Roxana, Ispir Petre Rzvan, Jenaru Constantin Mircea, Jianu Diana Florentina, Lazr Alexandru, Manafu Daria
Georgiana, Neagoe Andrei Daniel, Nistor Sara, Oranu Erica, Ptracu Alexandru, Petculescu Florin Cristian, Pistriu
Andrei Mircea, Pndici Andreea Denisa, Prvu Denisa, Popescu tefan Daniel, Rdulescu Octavian tefan, Srcin
Rzvan Constantin, Srbu Cosmin Constantin, meler Marian Cristian, Tudosie Ctlina Bianca, Vancea tefan Vlad,
Vian Cristian Mihai, Vlcu Rzvan, Vntoru Elena Emanuela (Prof. Draga Ttucu Mariana)

COALA GIMNAZIAL MUEREASCA DE SUS, JUD. VLCEA:


Clasa a V-a: Drgoescu Adina, Grmticioiu Gabriela, Andreescu Andreea (Prof. dr. Mihaela Albici)
Clasa a VI-a: Urea Raluca, Grmticioiu Ileana, Mihalcea Ionu, Gaic Izabela, Olaru Mdlin, Drgoescu
Delia, Urea Alexia, Margu Robert, Urea Alexia . Clasa a VII-a: Vlcu Roxana, Tutunaru Alexandru, Olaru
Ctlin, Ghierel Albert (Prof. dr. Mihaela Albici)
LICEUL TEHNOLOGIC DE TURISM DIN CALIMANESTI - VALCEA
Clasa a X-a: Rizea Ionela (Prof. Neacu Stelua)
Clasa a XI-a: Ceap Bianca Elena, Sora Florentina, ugulan Vlad (Prof. Neacu Stelua)
Clasa a XII-a: Gheorghion Florian (Prof. Neacu Stelua)
Clasa a X-a: Antonie Marcel, Ungureanu Daniela, Lupu Cristian (prof. Barbu Daniela)
Clasa a XI-a: Lupu Alexandru-Daniel (prof. Barbu Daniela)
Clasa a XII-a: Mate Andreea, Tanasoiu Raluca (prof. Barbu Daniela)
LICEUL CONSTANTIN BRNCOVEANU HOREZU VLCEA
Clasa a V-a: Rpeanu Ana Maria, Cpitanu Cristina, Sicoe Roxana, Crian Andrada, Ogrezeanu Valentin,
Manea Remus, Siulescu Ana Maria, Chivran Riana, Andrei Sbrina, Danciu Alexandru, Mormonea Bogdan,
Belciu Oana Elena, Tgr Denisa, Rizescu Delia, Dobrioiu Iulia, Brutaru George, Srdroiu Gabriel, Piu
Andreea, Cioranu tefan, Crstoiu Vlad, Gineric Raluca (Prof. Drghici Ana Maria);
Clasa a VI-a: Andreescu Andrei; Mitoi Cristina; Rpeanu Adrian; Trtreanu Andra; Matei Mdlina; Drghici
Miruna; Andronescu Sebastian,Ureche Andreea, Antonie Bogdan (Prof. Chievescu Tatiana)
Clasa a VII-a: Ghinea Mara, Anica Stefan, Popa tefania, Rachieru Razvan, Ion Stefan, Robu Nicoleta,
Spltoreasa Andrei, Viezuin Marius (Prof. Drghici Ana Maria) Cenusoiu Georgiana; Rada Alexandra;Stefan
Diana (Prof. Chievescu Tatiana), Giubega Maria Iustina, Buza Andrada (Prof. Cazacu Ersilia)
Clasa a VIII-a: Surcel Eduard (Prof. Drghici Ana Maria), Tioiu Teodor, Radu Andrei (Prof. Chievescu
Tatiana)
Clasa a V-a: Draghici Gabriel (Mazilu Marius, Colegiul National de Informatica "Matei Basarab", Rm.
Vlcea)

Probleme propuse_______________________________________________________________________________________ 58

Clasa a VIII-a: Drghici Andrei (Prof. Smrandoiu tefan, coala Gimnazial "Take Ionescu", Rm.
Vlcea)
C.T.T. AUTO
Clasa a XII-a: Bobleanta Madalin, Buca Claudiu, Bulea Madalin, Cratau Robert, Girtoi Alberto, Iordaiche Mirel,
Jianu Daniel, Oproiu Petre, Pristoleanu Gigel (prof. Maris Marinela)
C.GIMN. MIHAI EMINESCU CRAIOVA
Clasa I A: Olaru Alessia, Anua Eliza Daria, Dominte Alexandru (Prof. Cimpoeru Elena)
Clasa I B: Budoi Bianca, Grigorie Alisia, Mare Carina (Prof. Ionescu Mihaela)
Clasa I C: Roman Patrick, Kodreanu Andrei, Ionescu Andrei (Prof. Bododea Eugenia)
Clasa a II-a B: Macrea Maria, Florescu Iuliana, Raicu Emil (Prof. Crn Firua)
Clasa a IV-a A: Bana Denisa Elena, Brane Bianca Elena, Clin Ionu Andrei, Cruntu Rzvan Ionu,
Ciocazan Ariana Nicoleta, Cotea Elena Alexandra, Dan Sebastian, Duinea Anda Maria Cristina, Glug
Andreea Alexandra, Gofi Andreea Bianca, Kontogiorgis Mihalis Gabriel, Negricea Radu Gabriel, Petrescu
Ioana Ariana, Sakr Ahmed Ramy, Tugulan Ana- Maria, Vduva Rare Andrei, Vlduu Sergiu Andrei,
Voronechi Alina Elena, Iordache Andreean (Prof. Voican Victoria)

59 ______________________________________________________________________________ Probleme propuse

MINISTERUL EDUCATIEI, CERCETARII, TINERETULUI SI SPORTULUI


REVISTA DE MATEMATICA MINIMATH
Secretar general de redactie
Prof.Dr. Adrian Lupu
INSPECTORATUL SCOLAR JUDETEAN MEHEDINTI
COLEGIUL TEHNIC ,,DECEBAL DR. TR. SEVERIN
Societatea de Stiinte Matematice din Romania- Filiala MEHEDINTI
Presedinte Prof. Sitaru Daniel
Organizeaz
in colaborare cu :
INSPECTORATUL SCOLAR AL MUNICIPIULUI BUCURESTI
SCOALA GIMNAZIALA SF. VOIEVOZI, BUCURESTI
SC. GEN. NR. 24 BUCURESTI
INSPECTORATUL SCOLAR JUDETEAN DOLJ ,
SC. GEN. MIHAI EMINESCU, CRAIOVA
INSPECTORATUL SCOLAR JUDETEAN OLT ,
SC. GEN. NR. 1 SCORNICESTI
INSPECTORATUL SCOLAR JUDETEAN ARGES,
SCOALA GIMNAZIALA ,,NICOLAE SIMONIDE , PITESTI
INSPECTORATUL SCOLAR JUDETEAN GORJ
SCOALA GIMNAZIALA C.SAVOIU, TG. JIU
INSPECTORATUL SCOLAR JUDETEAN CARAS
SCOALA GIMNAZIALA NR. 9 RESITA
SCOALA GENERALA VUK KARADZICI, DONJI MILANOVAC , SERBIA
LICEUL BORISLAV PETROV BRACA , VRSAC , SERBIA
CONCURSUL DE MATEMATIC MINIMATH
EDITIA V
7 mai 2016

Concursul vizeaza cu precadere urmatoarele obiective:


Atragerea copiilor catre studiul matematicii.
Cresterea atractivitatii si accesibiltatii invatarii matematicii.
Familiarizarea elevilor cu rigorile unui examen.
Sprijinirea scolilor in vederea evaluarii corecte a nivelului de pregatire al elevilor la matematica.
Incurajarea cooperarii intre scoli si intre profesori pentru diversificarea si imbunatatirea metodelor si
mijloacelor didactice de predare si invatare la matematica.
REGULAMENTUL CONCURSULUI

Participanti: elevii claselor pregatitoare, I,a II -a , a III -a , a IV -a , V-VIII


Inscrierea participantilor:
-Perioada 14 ianuarie- 1 mai 2016
-Modalitati de inscriere:
mail: lga1973@yahoo.com , yasw01@yahoo.com
direct la oricare din redactorii revistei de matematica Minimath
-Nu se percepe taxa de inscriere

Probleme propuse_______________________________________________________________________________________ 60

Proba de concurs
7 mai 2016 (sambata) , orele 10.00-12.00
Proba de concurs consta in rezolvarea unui test de 4 probleme deosebite din revistele Minimath
corespunzatoare fiecarei clase si o cerinta optionala si suplimentara doar pentru cei care vor sa concureze
pentru obtinerea diplomelor de originalitate, fantezie matematica , rigoare matematica, cultura matematica si
perspicacitate matematica. Problemele vor fi selectate din culegerile de matematica Minimath si din revistele
de matematica Minimath din anul 2016 si anii anteriori.
Locul de desfasurare: La sediul fiecarei unitati scolare desemnate sa organizeze , la nivel judetean.
Programul concursului
- 8.00-9.15
sosirea si repartizarea in sali a participantilor
- 9.15-9.30
Cuvantul de deschidere online
- 9.30-10.00
distribuirea online a subiectelor
- 10.00-12.00
proba scrisa
- 12.30-15.30
corectarea lucrarilor
- 15.30 16.00
Festivitate de premiere
- 16.00
Cuvantul de La revedere pentru editia viitoare
Premii
Pentru fiecare clasa se vor acorda urmatoarele premii:
Locul I:
diploma + carti, reviste din colectiaMinimath
Locul II:
diploma + carti, reviste din colectiaMinimath
Locul III:
diploma + carti, reviste din colectiaMinimath
Mentiuni:
diploma + carti , reviste din colectiaMinimath
Toti elevii vor primi diploma de participare.
De asemenea , daca este cazul , se vor acorda indiferent de clasa la care au participat ,pentru elevii
care se disting prin originalitate, fantezie matematica , rigoare matematica, cultura matematica si
perspicacitate matematica diplomele de onoare: Lalescu , Titeica, Onicescu si Dimitrie Pompei.
Se va acorda la fiecare editie diploma MOISIL pentru cel mai entuziast profesor de matematica acelui dascal
care prin atitudine, competenta si daruire merita cu prisosinta recunoasterea organizatorilor.

61 ______________________________________________________________________________ Probleme propuse

MINISTERUL EDUCAIEI, CERCETRII , TINERETULUI SI SPORTULUI

INSPECTORATUL COLAR
AL JUDEULUI MEHEDINTI

INSPECTORATUL COLAR
AL MUNICIPIULUI BUCURETI

COLEGIUL TEHNICDECEBAL
Drobeta Turnu Severin

PRIMARY AND SECONDARYSCHOOL


Vuk Karadjici Donji Milanovac , Serbia

REVISTA DE MATEMATIC MINIMATH

SOCIETATEA DE TIINE MATEMATICE


FILIALA MEHEDINI

Secretar general de redacie


Prof. dr . Adrian Lupu

Preedinte Prof.Sitaru Daniel

SIMPOZION INTERNAIONAL ONLINE DE MATEMATIC


MINIMATH
EDIIA VII
Matematica colar - fundamentul unei coli moderne

7 mai 2016

Probleme propuse_______________________________________________________________________________________ 62

Datoria oricrei instituii colare din sistemul naional de nvmnt, indiferent de tip, nivel i form de
organizare a activitii, este de a asigura predarea, nvarea, i , acolo unde este cazul, cercetarea de calitate
pentru a contribui la dezvoltarea profesional i personal a elevilor i la bunstarea societii i dezvoltarea
sa durabil.
Asigurarea calitii educaiei matematice exprim capacitatea de a oferi programe de educaie, n
conformitate cu standardele nationale . Asigurarea calitii n educaia matematic este un demers dinamic ,
ce presupune implicare i responsabilizare, pe de o parte, precum i gndire strategic i control, pe de alt
parte.
SCOPUL SIMPOZIONULUI:
Simpozionul constituie un bun prilej pentru sporirea nivelului preocuprilor privind creterea calitii
educaiei matematice n coala romneasc.
OBIECTIVELE SIMPOZIONULUI
Analiza comparativ a diferitelor culturi organizaionale i a percepiilor asociate conceptului
de calitate a educaiei matematice.
Promovarea strategiilor privind asigurarea educaiei matematice de calitate n coala
romneasc la nivelul standardelor europene.
Educarea participanilor n spiritul asigurrii calitii educaiei matematice.
Identificarea de propuneri pentru mbuntirea politicilor i a strategiilor educaionale
privitoare la asigurarea calitii educaiei matematice.
Rectigarea ncrederii n coal i n educaia matematic de calitate ca mijloace de
dezvoltare personal, de integrare i de promovare social.
Diseminarea exemplelor de bun practic n privina asigurrii calitii educaiei matematice.
GRUPUL INT:
Cadre didactice (educatori, nvtori/institutori, profesori), specialiti , consilieri colari, manageri educaionali,
prini i elevi.
SECIUNILE SIMPOZIONULUI:
Seciunea S1 : Actualiti n matematica pentru ciclul primar
Seciunea S2 : Actualiti n matematica pentru ciclul gimnazial
Seciunea S3 : Actualiti n matematica pentru ciclul liceal
Seciunea S4 : Impactul matematicii asupra altor tiine
Seciunea S5 : Abordri n matematica colar din alte ri
COORDONATORII SIMPOZIONULUI:
1. Prof.dr. Adrian Lupu, CT Decebal, Dr. Tr. Severin
2. Inst.Corina Andrei, c.Gen. Nr. , Bucureti.
3. Prof. Danijel Dragas , Primary and Secondary School Vuk Karadjici,Donji Milanovac , Serbia
4. Prof.dr. Brndua Nicolaescu Ovesea, CT Edmond Nicolau, Bucureti
5. Prof. Drd. Sanja Nicolic , Primary and Secondary School Vuk Karadjici,Donji Milanovac ,
Serbia
6. Prof.Doina Ardeljan , High School Borislav Petrov Braca, Vrsac, Serbia
Membri onorifici ai comitetului de organizare:
1.
Prof. Dr. Ciniceanu George , Colegiul Naional Traian , Dr. Tr. Severin
2.
Prof. Dr. Prajea Manuela , Colegiul Naional Traian , Dr. Tr. Severin
3.
Prof. Dr. Stretcu Daniel , Colegiul Naional Gh. ieica, Dr. Tr. Severin
4.
Prof. Daniel Nnui , Inspector de matematic la ISJ Mehedinti
Echipa de organizare a activitatilor
Echipa de organizare a activitatilor
1. Pentru seciunea S1:
Inst. Srebranca Mateiaevici, Moldova Nou; inst. Elena Crsta, Reia; inst. Costa Moatr, Reia;
inst.Victor Grindeanu, Reia, inst. Oana Nicoleta Crstoiu, Timioara, inst. Aurelia Ghidel , Drobeta Turnu
Severin, inst. Corina Andrei, Bucureti; Prof.Inv. Primar Copaci Ileana , Strehaia, ; Prof.Inv. Primar Mirela

63 ______________________________________________________________________________ Probleme propuse

Bolbotina , Baile Herculane , ; Prof.Inv. Primar Liana Carciumaru , Tg. Jiu, Prof. Inv. Primar Stoian Violeta
, Bucuresti, Prof.Inv. PrimarGaman Camelia , Dr. Tr. Severin , Prof.Inv. Primar Ioana Zahan , Cluj Napoca
,
2. Pentru seciunea S2:
prof. Bolbotina Constantin, Baile Gerculane, prof.dr.Natalia-Lavinia Mogonea, Cluj-Napoca, prof.Molnr
Zsolt, Cluj-Napoca, prof. Dorel Cpraru, prof. Drobeta Turnu Severin, Marica Pi-Rada, Drobeta Turnu
Severin, prof. Petrua Lpdat, prof. Anghel Bdlu, Dina , Bosneagu , Drobeta Turnu Severin, prof.
Tatucu Mariana, Drobeta Turnu Severin, prof. Veronica Riza, Orioara, prof.Elena Nicolae , Craiova ,
prof. Bosneagu Dina , Dr. Tr. Severin, prof. Mircea Tudosie , Strehaia,prof. Elena Ramniceanu , Dr. Tr.
Severin, prof.Ionic Constantin, Drobeta Turnu Severin, prof. Gheorghe Calafeteanu, Drobeta Turnu
Severin, prof.Rodica Popescu ,Drobeta Turnu Severin, Prof. Valentin Radulescu , Scornicesti
3. Pentru seciunea S3:
Prof. Daniela Barbu , prof. Steluta Neacsu , Rm . Valcea prof. Gina Tudosie, prof. Strehaia, prof. Teodor
Deaconu, Reia, prof. Ovidiu Ticui, Drobeta Turnu Severin, prof. Doru Preneanu, Drobeta Turnu
Severin, prof. Dan Nedeianu, Drobeta Turnu Severin, prof. Laura Amariei , Galati, prof. Naidin Delia ,
Caracal-Inspector de matematic la ISJ Olt,Doctor in matematica . Mihai Babutia, , Timisoara, Prof. Sitaru
Daniel , Dr. Tr. Severin
4. Pentru seciunea S4:
Prof. Soare Iuliana , c. Gen . Nr.1 , Bucureti, Prof. Silvia Cru , Craiova
5. Pentru seciunea S5: toi redactorii revistei Minimath din strintate
prof.Doina Ardelean, Vrset, Serbia, Masteranda Nicoleta Loredana Ularu Condescu, Timioara, Justine
Andrea Secara, Master of Teaching (Sec), Melbourne, Australia , prof. Mioara Vagaon , Toronto, Canada,
prof.Stefan Strandel , Kungsbacka , Suedia , prof. Maria Letizia Vitale , Caserta , Italia , prof. David Batt ,
Londra , Marea Britanie , prof. Marisa Lucianetti , prof. Floriana Agostinello , prof.Daniela Capello, prof.
Victoria Canistra , prof. Raffaella Basile , prof. Simona Travisano, Varese Italia , prof. Tani Toshikazu,
Hokkaido , Japan, prof. Srdjan Ognjanovic , Belgrad
LOCUL DE DESFURARE:
Site-ul de desfasurare este www.simpozion2016.minimath.ro
PROGRAMUL SIMPOZIONULUI
Smbt, 7 mai 2016, ora 9.30
FESTIVITATEA DE DESCHIDERE A SIMPOZIONULUI, IN CADRUL CREIA VOR FI PREZENTATE , SUCCESIV,
PE SITE-UL ANUNAT:
9 Cuvntul de deschidere al organizatorilor.
9 Alocuiunile reprezentanilor Ministerului Educaiei , ISMB ,ISJ Mehedini , Olt , Teleorman , Dolj,
Timis , Cara Severin , Braov , Casei Corpului Didactic Caras.
9 Prezentarea alocuiunilor de ctre invitaii din strintate.
9 Prezentarea lucrrilor participanilor,urmat de dezbateri online pe forum , pe fiecare din sectiunile
stabilite
9 Decernarea diplomelor de participare, pe adresele de mail ale participantilor
PROBLEME ORGANIZATORICE
Condiii pentru tehnoredactare
9 lucrrile vor fi tehnoredactate n Microsoft Word,
9 lucrarea va fi nsoit de un rezumat ce nu va depi o pagina , cu specificarea bibliografiei folosite n
redactarea lucrrii daca este cazul.
9 Lucrarea va fi insotita (dupa caz) de o prezentare video de maxim 3 minute realizata de cei care
doresc acest lucru; detaliile vor fi puse la dispozitie de catre organizatori.
NOT: Organizatorii nu ateapt doar referate teoretice ci mai degraba materiale practice care sa ilustreze
activitatea la clas , experiena didactic sau chiar rezultatele activitaii de cercetare n acest domeniu
Termenul limit de depunere/ trimitere a lucrrilor : 1 mai 2016
Lucrarea n forma ei final, rezumatul acesteia, fia de nscriere si prezentarea video (daca este cazul) se vor
trimite ca fiiere ataate (*.doc) pe adresa de e-mail: simpozionminimath2016@yahoo.com , specificand
explicit la care din seciunile prestabilite se dorete inscrierea.

Probleme propuse_______________________________________________________________________________________ 64

Participanii care trimit rezumatul lucrrii de simpozion pn la data specificat mai sus vor primi , n format
electronic, pe mail, un CD cu I.S.B.N , precum i programul oficial al simpozionului, avnd inserate toate
titlurile lucrrilor prezentate
NU SE PERCEPE TAX DE PARTICIPARE LA SIMPOZION.
nscrierea se va face pn la data de 1 mai
2016 doar prin e-mail , pe adresa :
simpozionminimath2014@yahoo.com
Evaluare
9 Diplome pentru participani
9 Diplome pentru colile/grdiniele cu numr mare de participani;
9 Articole n reviste;
9 Postere;
9 Album foto;
9 Portofoliul simpozionului;
9 CD-ul simpozionului;
Persoane de contact:
Prof. dr. Adrian Lupu 0722408338 , lga1973@yahoo.com
Prof. inv. primar. Corina Andrei 0723291088 , yasw01@yahoo.com

You might also like